Figure 6.1 We are able to think, move, and eat because of collections of neurons in our bodies. This photograph shows the neurons of a ground squirrel.

images

Preview

“Among all the mathematical disciplines the theory of differential equations is the most important ... It furnishes the explanation of all those elementary manifestations of nature which involve time.”

Sophus Lie, Leipziger Berichte, 47 (1805)

Equations containing one independent and one dependent variable, as well as the derivative of the dependent variable with respect to the independent variable, are known as first-order ordinary differential equations (or first-order ODEs for short). An example of a first-order ODE where the independent variable is x and the dependent variable y is:

images

The right hand side of this differential equation depends on t and y and, consequently, is known as a non-autonomous differential equation. When the right hand side only depends on the dependent variable, the equation is autonomous. In this Chapter, we develop quantitative and qualitative methods to understand solutions of these equations. A solution is a function y(t) that satisfies the equation in question over a specified interval of time.

Despite their moniker, ODEs are anything but “ordinary.” They have been used to describe extraordinary things such as world population growth, the dynamics of nerve impulses, and the spread of diseases. For instance, in Example 5 of Section 6.6, we use differential equations to explore how populations of neurons can store memories. In this chapter, we study these extraordinary equations three ways. First, after introducing some basic terminology and models, we will derive analytical solutions for special types of ODEs using integration techniques. Second, as ODEs often cannot be solved explicitly, we will introduce techniques that shed light on the qualitative behavior of ODEs. Third, we will use technology to generate and visualize numerical solutions to these ODEs. To this end, we will discuss a numerical method, namely Euler's method, to generate specific solutions to ODEs. By no means will this discussion be exhaustive. This chapter and Chapter 8 provide only a taste of this powerful mathematical formulation, which has been used extensively to understand the dynamics of biological systems.

6.1 A Modeling Introduction to Differential Equations

Differential equations describe how quantities change continuously over time. One of the first applications of differential equations to biology was to population growth: understanding how the sizes of populations–whether they be viruses, plants, or animals–vary in time. We begin our study of differential equations by examining some of these applications.

Exponential population growth and decay

At the beginning of the twentieth century, several notable biologists, including Georgyi F. Gause and T. Carlson, studied population growth of yeast. Both biologists grew yeast under constant environmental conditions in test tubes and flasks as illustrated in Figure 6.2, and they regularly monitored the densities. The resulting data from Carlson's experiment are shown in Figure 6.3.

images

Figure 6.2 Density of yeast growing in a test tube a and a flask b configured so that the same amount of medium can have two different levels of exposure to the air.

Data Source: G. F. Gause, “The Struggle for Existence.” This is a book published in 1934 by Williams and Wilkins, Baltimore.

images

Figure 6.3 Carlson yeast data plotted hourly over an eighteen-hour period.

Data Source:Über Geschwindigkeit und Grösse der Hefevermehrung in Würze,” Biochem. Z.57 (1913): 313–334.

The principle entia non sunt multiplicanda praeter necessitatem (i.e., entities must not be multiplied beyond necessity) is attributed to Franciscan friar William of Ockham (ca. 1287–1347).

One of the goals of this section is to come up with a model that describes Carlson's yeast data. In developing the model, we adhere to the principle of parsimony, an operational principle used in science that requires beginning with the simplest model and only add more complexity as needed. It is also known as Occam's razor or more simply as the KISS principle–Keep It Simple, Stupid! Thus, we begin by formulating and analyzing the simplest possible model and introduce elaborations only as necessary. We start by modeling the initial growth phase of a relatively simple population: the number of cells in a yeast culture.

Example 1 Constant per capita growth rate model

The growth of a population of size N at time t, denoted by the function N(t), is determined by four processes: birth, death, immigration, and emigration. The simplest model follows from these assumptions:

  • The system is closed: There is no immigration or emigration.
  • Birth rates are proportional to the population density: The more individuals there are, the more births there are. If b is the proportionality constant, then the birth rate is b N(t) and b is called the per capita birth rate.
  • Death rates are proportional to the population density: The more individuals there are, the more deaths there are. If d is the proportionality constant, then the total death rate is dN(t) and d is called the per capita death rate.

Write a differential equation model that embodies these assumptions.

Solution Let N denote the population density and t time. Under the stated assumptions, the model is as follows:

images

Here, the per capita birth rate minus per capita death rate, R = bd, is referred to as the intrinsic growth rate or the instantaneous per capita growth rate because R = images for N > 0.

A solution of the equation, images = RN, is a function N such that

images

We will analyze the solution of this differential equation qualitatively and analytically. A qualitative analysis involves discovering the qualitative behavior of solutions. In other words, it involves determining whether the solutions are increasing, decreasing, remaining constant, or even oscillating, without worrying about the exact form of the solution.

Example 2 Qualitative behavior of the constant per capita growth rate model

Consider the growth of a population modeled by

images

where the initial value of the population N(0) at time t = 0 is positive. Discuss how the behavior of the population depends on the sign of R.

Solution

Case R = 0. Since N′(t) = RN = 0 for all t, the rate of change in N is identically zero. Hence, the population density N(t) stays equal to its initial value N(t) = N(0) for all t > 0.

Case R > 0. In this case N′(t) = RN > 0 for all t. Therefore, the population density increases indefinitely.

Case R < 0. In this case N′(t) = RN < 0 for all t. Therefore, the population density decreases indefinitely.

The three qualitative cases in this example correspond to three regimes of population behavior:

Constancy: R = 0 and the per capita birth rate b equals the per capita death rate d. The population neither grows nor declines.

Growth: R > 0 and the per capita birth rate b exceeds the per capita death rate d. The population increases over time.

Decay: R < 0 and the per capita death rate d exceeds the per capita birth rate b. The population decreases over time.

We made all of these qualitative predictions by looking at the sign of the right-hand side of N′ = RN. General qualitative methods for making these predictions are discussed further in Sections 6.4 and 6.5.

In contrast to a qualitative analysis, an analytical approach involves finding explicit solutions to differential equation models. For this constant per capita growth rate model, finding an analytical solution means finding a function N(t) such that its derivative is R times itself; that is, N′(t) = RN(t).

Example 3 Exponential growth model

Verify that the function

images

is a solution to the differential equation

images

for any value C. Provide an interpretation for C and for the behavior of N(t) when R > 0 and R < 0.

Solution To verify that N(t) = CeRt is a solution, it suffices to plug N(t) into both sides of the equation images = RN:

images

Thus, N(t) = CeRt is a solution for any constant C. Evaluating N(t) at t = 0 gives N(0) = CeR×0 = C. Hence, C represents the initial population density and, population models must satisfy C ≥ 0. Furthermore, because N(t) is an exponential function, we have:

Exponential growth: For R > 0, the population density N(0)eRt increases exponentially to infinity over time.

Exponential decay: For R < 0, the population density N(0)eRt declines at an exponential rate, asymptotically approaching zero.

Using the solution N(t) = CeRt, we can determine how well our simple model N′(t) = RN fits the initial phase of Carlson's yeast data. We can either fit the model directly or use a semi-log plot to fit a linear model through the data since, by taking logarithms, our exponential growth model N(t) = N(0)ert can be written as

images

where k = ln C = ln N(0).

Example 4 Carlson's data: exponential growth

Yeast provide an ideal organism for population growth studies because they are easy to culture in liquid media, and the number of individuals per unit volume can be counted under a microscope, as depicted in Figure 6.4 where some cells can be seen reproducing by budding. The data that Carlson gathered in his yeast culturing study are show in Table 6.1.

images

Figure 6.4 Yeast cells, as seen under a microscope.

Table 6.1 Population densities (number/unit volume) for a growing yeast culture at one-hour intervals

images

As illustrated in Figure 6.3, the initial phase of population growth appears to be exponential.

  1. Use the data to estimate the parameters C and R in the growth model N(t) = CeRt, where t is measured in hours, over the first three hours; then plot the model together with the data over the first eight hours to see how well the model continues to fit.
  2. Use the logarithms of the data to estimate the value of R that provides the best-fitting line ln N(t) = Rt + ln N(0) to a semi-log plot of the first eight hours of data.

Solution

  1. Recall, C represents the initial population density, so C = N(0). Hence, C = 9.6. To estimate R over the first three hours, we choose the data point N(3) = 47.2 (bearing in mind that a different data point would yield a similar, but different graph) and solve

    images

    Since the time is in hours, R ≈ 0.53 per hour over the first three hours.

    images

    Figure 6.5 Exponential model (solid line) fitted to the first three hours of the Carlson yeast data (dots).

    A plot of N(t) = 9.6e0.53t against the data is shown in Figure 6.5. The equation we derived fits the data well until t = 6. After t = 6, the equation overestimates the population size.

  2. Using the approach laid out in Example 7 of Section 4.3, we obtain the following table:

    images

    Now we seek to minimize the function

    images

    Differentiating S with respect to R yields

    images

    Setting S′(R) = 0 and solving for R yields

    images

    which is somewhat lower than the R = 0.53 that we estimated directly from the data over the first three hours. The semi-log regression line is

    images

    A plot of this line is provided in Figure 6.6.

    It is not surprising that we obtained a lower value for R using a semi-log regression for the first eight hours of data, as our exponential model in part a overestimates the growth rate from hours 6 to 8.

images

Figure 6.6 Semi-log regression plot (solid line) of exponential growth model through the first eight hours of the Carlson yeast data (dots).

Using the exponential model of growth, we can estimate the doubling time for a yeast population during the early stages of colony growth.

Example 5 Doubling time for yeast

For a population satisfying the equation

images

find the time in hours for the population to double.

Solution The solution of this differential equation is of the form

images

where C corresponds to the initial population density. To find the doubling time in hours, we need to find t such that N(t) = 2N(0) = 2C. Hence, we need to solve

images

The doubling time is about 1 hour and 18 minutes. This conclusion is consistent with the Carlson data during the first few hours of growth. For instance, we can see in Figure 6.5 that after 3 × 1.31 ≈ 4 hours, the yeast density has increased approximately by a factor of 23 = 8.

As you will see in Problem Set 6.1 and in Section 6.3, the exponential growth model dN/dt = RN is used to model radioactive decay, decay of drugs in the bloodstream, and decay of the number of viral particles in the blood of an individual treated with antiviral drugs.

Logistic growth

Although the exponential model provides a reasonable fit for the initial growth of the yeast population considered in the previous example, it substantially overestimates the population density during the seventh and eighth hours. Moreover, beyond eight hours, the data plotted in Figure 6.7 indicate that the yeast culture asymptotically approaches a density of around 660, while the exponential growth model exhibits unbounded growth. This phenomenon of decreasing per capita growth rate with increasing population density was first elaborated by Thomas Malthus (1766–1834) in his 1798 treatise “An Essay on the Principle of Population Growth.” Malthus recognized that as populations get larger, their per capita growth rate declines due to limited resources and interference among individuals. To deal with these limitations, we modify our model using the principle of parsimony.

images

Figure 6.7 Per capita growth rate as a function of density for the Carlson yeast data.

Using the Carlson data in Table 6.1, we estimated the per capita growth rate, R, of yeast as a function of population density, N. These estimates are plotted in Figure 6.7 and show that the per capita growth rate is a decreasing function. The exact form of R as a function of N is not uniquely determined. Following the parsimony principle, we begin with the simplest decreasing function of N with positive intercept on the R axis, which is the linear function. Let K denote the horizontal intercept and r the vertical intercept of this linear function. In other words, we choose the per capita growth rate R(N) in the model

images

to be the linear function

images

This equation is the logistic equation, which is arguably the single most important equation in population ecology. From the data plotted in Figure 6.7, we might guess that the vertical intercept r lies somewhere between 0.5 and 0.6 and that the horizontal intercept K lies somewhere between 600 and 700. For reasons that become clearer in the next example, the value N = K is called the carrying capacity for the population. The parameter r is called the intrinsic growth rate.

Logistic Equation

If in the equation

images

the instantaneous per capita growth rate function R(N) has the form

images

then we obtain the logistic equation

images

The parameter r > 0 is the intrinsic growth rate and the parameter K > 0 is the carrying capacity. The parameters r and K are, respectively, the vertical and horizontal intercepts of the graph of the function R(N).

What can we say about the behavior of the solutions to the logistic equation? We partially answer this question with a qualitative analysis. Finding explicit solutions have to wait until the next section.

Example 6 Qualitative analysis of the logistic equation

Assuming that r > 0 and K > 0, describe qualitatively how solutions to the logistic equation depend on the initial value of N.

Solution Qualitatively there are three types of solutions when initially N ≥ 0.

Equilibrium solutions: If N = 0 initially, then the population growth rate images = r × 0(1 − 0/K) = 0 initially and the population density continues to remain at zero. We verify in the next section that N = 0 for all t is a solution to this differential equation. Similarly, if N = K initially, then images = rK(1 − K/K) = 0 initially and the population density continues to remain at K. These unchanging (i.e., constant) solutions are called equilibrium solutions and are illustrated in Figure 6.8a.

images

Figure 6.8 Different solutions for the logistic equation.

Increasing and saturating: If 0 < N < K, then rN(1 − N/K) > 0 and the population growth rate images is positive. For a population starting between 0 and K, we expect the population density to increase. However, since images gets close to zero as N gets close to K, we expect the population to increase less rapidly as it approaches K and to asymptotically saturate at K, as illustrated in Figure 6.8b. We will show this formally in the next section where we find explicit solutions to the logistic equation.

Decreasing and saturating: If N > K, then rN(1 − N/K) < 0 and the population growth rate images is negative. In this case the population density declines over time. As images becomes less negative as N approaches K from above, we expect population density to decline less rapidly as it approaches K and the population density to asymptotically level off at K. This is illustrated in Figure 6.8c.

Hence, as long as N > 0 initially, we expect the population density to approach the carrying capacity K of the environment, as will be seen to be true once we have solved the logistic equation in the next section.

Example 7 Logistic model for the yeast data

Parameterize the logistic model for the Carlson yeast data given in Example 4.

Solution The Carlson data in Table 6.1 suggest that the population density is approaching an asymptotic value of 660. Hence, we choose K = 660. To estimate r, notice that when N is small

images

In other words, at low densities we expect to see approximately exponential growth. Using our work from Example 4, we set r = 0.53. Thus, the specific logistic equation in this case is

images

In the next section we derive the solution to the logistic equation that we can compare directly to the data. Problem Set 6.1 has examples in which the logistic model describes the spread of AIDS and the use of the iPhone in the United States.

External influences on populations

In addition to understanding the feedbacks of populations on their own growth, it is useful to account for external influences on these populations. To do this, we need to extend the models to incorporate elements not included in the initial model. In the 1970s the biomathematician Colin W. Clark at the University of British Columbia, building on the work of others, invented a new field of research at the intersection of mathematical population biology and economics, which he called mathematical bio-economics.* His work extended the logistic equation to account for the economics of harvesting biological populations. The most important applications were in the whaling and fisheries industries. Clark's analysis is based on logistically growing populations that are harvested at a rate h(N) over time:

images

Two cases are of particular interest are:

  • Constant harvesting: In this case, the harvest rate is h(N) = h for all N > 0. If harvesting drives N to 0, as it has in some real populations, then h(N) = 0 for N = 0.
  • Proportional harvesting: In this case, the harvest rate is h(N) = vN, where the constant of proportionality v > 0 is called the harvesting effort variable.

Example 8 Harvesting queen conchs

Consider a population of queen conchs in the Bahamas that, in the absence of harvesting, exhibit logistic growth. Let N represent the number of conchs in a well-defined area and t be measured in years. For ease of computation, assume that the intrinsic growth rate of this population is r = 10 and that the carrying capacity of the area in which the conchs are located is K = 10,000 individuals.

images

Figure 6.9 Conchs are harvested for home reef aquariums, as well as for their beautiful shells.

  1. Write a logistic harvesting model for the case where 21,000 individuals are removed from the population every year.
  2. Determine qualitatively the fate of the population and how it depends on the initial number of conchs in the population.
  3. Discuss what happens if the harvesting rate is h = 30,000 conchs per year.

Solution

  1. Since we are harvesting at a constant rate of 21,000 individuals per year, the model is

    images

  2. The qualitative analysis reduces to understanding for what values of N is

    images

    Case I: images = 0.

    images

    These are the equilibrium values, that is, values where images = 0.

    Case II: images < 0. From our work in case I, we see that images < 0 if N < 3,000 or N > 7,000. Hence, if N > 7,000, then the population would decrease, but decrease more slowly as N approaches 7,000 (i.e., images is close to zero for N near 7,000). Consequently, if N > 7,000, we would expect the population to decrease and to saturate at 7,000 (we say expect, because the notions expressed here can only be made more precise once we have considered additional theory). Alternatively, if N < 3,000, the population would continually decrease to 0 (i.e., extinction) as images becomes more and more negative as N continues to decrease.

    Case III: images > 0. From our work in case I, we see that images > 0 if 3,000 < N < 7,000. We would expect the population to increase, increase more slowly as it approaches 7,000, and to saturate at 7,000.

  3. First we note that the conch growth rate

    images

    has a maximum at N = 5,000 with

    images

    Thus, the right-hand side of the conch growth equation subject to a harvesting rate of 30,000 conchs per year satisfies

    images

    Hence, harvesting the population at this rate will drive it rapidly to extinction.

PROBLEM SET 6.1

Level 1 DRILL PROBLEMS

Write a differential equation to model the situation in Problems 1 to 8. Do not solve.

1. The number of bacteria in a culture grows at a rate that is proportional to the number of bacteria present.

2. A sample of radium decays at a rate that is proportional to the amount of radium present in the sample.

3. In Section 6.3, we will introduce Newton's law of cooling. Newton's law states that the rate at which the temperature of a body changes is proportional to the difference between the body's temperature T and the ambient temperature A.

4. In Section 6.3, we will study the von Bertalanffy growth equation. As part of that study, we will formulate a differential equation which states that the rate at which the mass, M, of a healthy critter grows through absorption of food is directly proportional to its surface area L2 and declines through respiration at a rate proportional to its mass L3. If M is proportional to L3—that is, M = kL3 for some positive constant k—then write the model in terms of the unknown function M.

5. According to Benjamin Gompertz (1779–1865), the growth rate of a population is proportional to the number of individuals present, where the factor of proportionality is an exponentially decreasing function of time.

6. When a person is asked to recall a set of N facts, the rate at which the facts are recalled is proportional to the number of relevant facts in the person's memory that have not yet been recalled.

7. The rate at which an epidemic spreads through a community of P susceptible people is proportional to the product of the number of people y who have caught the disease and the number Py who have not.

8. The rate at which people are implicated in a government scandal is proportional to the product of the number N of people already implicated and the number of people involved who have not yet been implicated.

For Problems 9 to 14 find the time for the population to double or halve its initial level, as appropriate.

images

images

15. How long does it take for a population to quadruple if its growth over years is modeled by

images

16. How long does it take for a population to quintuple if its growth over years is modeled by

images

17. How long does it take for the level of radioactivity to decay to 10% of its current level, if the level of radioactivity over years is modeled by

images

18. How long does it take for the level of radioactivity to decay to 1% of its current level, if the level of radioactivity over years is modeled by

images

A population model for Problems 19 to 22 is given by

images

where P(t) denotes population density at time t.

19. For what values is the population at equilibrium?

20. For what values is images > 0?

21. For what values is images < 0?

22. Describe how the fate of the population depends on the initial density.

A population model for Problems 23 to 26 is given by

images

where P(t) denotes population density at time t.

23. For what values is the population at equilibrium?

24. For what values is images > 0?

25. For what values is images < 0?

26. Describe how the fate of the population depends on the initial density.

27. Find the equilibria to the model images

Level 2 APPLIED AND THEORY PROBLEMS

Radioactive decay: Certain types of atoms (e.g., carbon-14, xenon-133, lead-210) are inherently unstable. They exhibit random transitions to a different atom while emitting radiation in the process. Based on experimental evidence, the number, N, of atoms in a radioactive substance can be described by the equation

images

where t is measured in years and λ > 0 is known as the decay constant. The decay constant is found experimentally by measuring the half-life, τ, of the radioactive substance (i.e., the time it takes for half of the substance to decay). Use this information in Problems 28 to 32.

28. Find a solution to the decay equation assuming that N(0) = N0.

29. For xenon-133, the half-life is five days. Find λ. Assume t is measured in days.

30. For carbon-14, the half-life is 5,568 years. Find the decay constant λ. Assume t is measured in years.

31. How old is a piece of human bone which contains just 60% of the amount of carbon-14 expected in a sample of bone from a living person? Assume the half-life of carbon-14 is 5,568 years.

32. The Dead Sea Scrolls were written on parchment at about 100 BC. Given that the half-life of carbon-14 is 5,568 years, what percentage of carbon-14 originally contained in the parchment remained when the scrolls were discovered in 1947?

33. King Arthur's Round Table. In Winchester castle there hangs a wooden round table, 18 feet in diameter and divided into twenty-five sections, one for the king and twenty-four for the knights. Some speculate that the Winchester round table is King Arthur's round table from the fifth century.* We know that the round table has been at Winchester since the fifteenth century. John Harding says in his chronical (1484) that the round table “ended at Winchester, and there it hangs still.” To put an end to the speculation regarding the Winchester round table, in 1976 it was taken down from the wall and tests were employed to determine the date of origin. The rate of decay of carbon-14 in the table (i.e., in dead wood) was found to be 6.08 atoms per minute per gram of sample. Estimate the age of the table to determine whether the Winchester table was King Arthur's round table. Hint: Use the facts that the half-life of carbon-14 in dead wood is 5,568 years and that in living wood the rate of decay of carbon-14 is 6.68 atoms per minute per gram of wood.

34. images

images

The Shroud of Turin is a rectangular linen cloth kept in the Chapel of the Holy Shroud in the cathedral of St. John the Baptist in Turin, Italy. It shows the image of a man whose wounds correspond with the biblical accounts of the crucifixion of Jesus Christ. In 1389, Pierre d'Arcis, the Bishop of Troyes, wrote a memo to the pope accusing a colleague of passing off a certain cloth, cunningly painted, as the burial shroud of Jesus Christ. Despite this early testimony of forgery, the Shroud of Turin has survived as a famous relic. In 1988, a small sample of the Shroud of Turin was taken and scientists from Oxford University, the University of Arizona, and the Swiss Federal Institute of Technology were permitted to test it. Suppose the cloth contained 92.3% of the original amount of carbon. Use this information to determine the age of the shroud.

35. Consider the queen conch logistic growth model presented in Example 8 with a general harvesting function h(N):

images

  1. Describe the qualitative behavior of solutions to this equation (i.e., the long-term abundance of the population) when h(N) = 25,000 for all N.
  2. Describe the qualitative behavior of solutions to this equation (i.e., the long-term abundance of the population) when h(N) = 5N for all N.
  3. Describe the qualitative behavior of solutions to this equation (i.e., the long-term abundance of the population) when h(N) = 12N for all N.

36. The cane toad (Bufo marinus) was introduced to Australia by the sugar cane industry to control two pests of sugar cane: the grey-backed cane beetle and the frenchie beetle.* Just over a hundred toads were brought to Gordonvale (near Edmonton, North Queensland) in 1935 and released into the cane fields. Unfortunately, due to an asynchrony between the life cycles of the cane toad and the sugar cane pests, the cane toad did not help suppress the cane beetle and the frenchie beetle. However, the cane toads ate almost everything else and grew at a tremendous pace. Now the cane toad is a major pest in Australia. The data in Table 6.2 describe the extent of the area occupied by the cane toads as a function of time.

A simple model to describe these data is given by

images

where A(t) is the area occupied at time t(years).

  1. Find a solution to this model such that A(0) = 32,800 and A(10) = 73,600. Estimate the parameter R.
  2. Estimate the area occupied by cane toads in 2004.
  3. Modify this model to account for removing cane toads at a rate Hkm2/yr beginning in the year 2004. Determine how large H needs to be to ensure that A starts decreasing.

Table 6.2 Area occupied (in square kilometers) by the cane toad in Australia

Year Area
1939 32,800
1944 55,800
1949 73,600
1954 138,000
1964 257,000
1969 301,000
1974 584,000

37. Consider the following problem of historical curiosity. The percentage of U.S. households that own a videocassette recorder (VCR) rose steadily from the time of their introduction in the late 1970s to the point at which other technologies displaced them. Let y(t) denote the percentage of U.S. households with a VCR where t is measured in years from 1980 to 1991.

images

Assume that

images

can be used to describe the data.

  1. Use the first and third data points and the approximation imagesry when y is small compared with K to estimate r.
  2. Use the fact that the data are saturating to estimate the value of K.

38. In the previous example, compare an estimate obtained for r using growth from 1981 to 1982 versus 1981 to 1984.

39. The Ohio Department of Health released the following data tallying the number of newly diagnosed cases of AIDS in the state from the initial stages of the epidemic to the early 1990s when the first antiretroviral drugs began to become widely available: (Cincinnati Enquirer, December 11, 1994).

images

Let y(t) denote the number of AIDS cases in Ohio in year t. Assume that

images

can be used to describe the data.

  1. Using the first few data points and the fact that imagesry when y is small, estimate r.
  2. Estimate the value of K.

40. Hyperthyroidism is caused by growth of tumor-like cells that secrete thyroid hormones in excess of normal amounts. If untreated, an individual with hyperthyroidism may experience extreme weight loss, anorexia, muscle weakness, heart disease, and intolerance to stress and eventually may die. The most successful and least invasive treatment option is generally considered to be radioactive iodine-131 therapy. This involves the injection of a small amount of radioactive iodine into the body. For the type of hyperthyroidism called Graves' disease, it is usual for about 40% to 80% of the administered radioactive iodine to concentrate in the thyroid gland. For functioning adenomas (“hot nodules”), the uptake is closer to 20% to 30%. Excess iodine-131 is excreted rapidly by the kidneys. The quantity of radioiodine used to treat hyperthyroidism is not enough to injure any tissue except the thyroid, which slowly shrinks over a matter of weeks to months. Radioactive iodine is either swallowed in a capsule or sipped in solution through a straw. A typical dose is 5 to 15 millicuries. The half-life of iodine-131 is eight days.

  1. Suppose that it takes forty-eight hours for a shipment of iodine-131 to reach a hospital. How much of the initial amount shipped is left once it arrives at the hospital?
  2. Suppose a patient is given a dose of 10 millicuries, of which 30% concentrates in the thyroid gland. How much is left one week later?
  3. Suppose a patient is given a dose of 10 millicuries, of which 30% concentrates in the thyroid gland. How much is left thirty days later?

6.2 Solutions and Separable Equations

In this chapter, we consider differential equations in which the right-hand side may explicitly depend on both the dependent and independent variables. If t is the independent variable and y is the dependent variable, then the equations we are considering have the general form

images

where h is an expression involving t and y. Although this moves us into the realm of functions of two variables, we postpone a more formal development of the theory for such functions to Chapter 8. Even though h(t, y) is a function of two variables, the solution y(t) to the differential equation is a function of the single variable t.

In this section, we discuss what it means to be a solution to these differential equations and develop an important method, separation of variables, for solving differential equations of the form images. Using this method, we explore super-exponential growth and logistic growth of populations.

Solutions to differential equations

A function y(t) is a solution to a differential equation if, when the function y(t) is substituted into both sides of the differential equation, the differential equation is satisfied.

Example 1 Verifying a function is a solution to a differential equation

Consider the differential equation

images

which is defined for all t ≠ −1. Which of the following functions are solutions for all t ≠ −1?

images

Solution

  1. To verify whether y(t) = t + 7 is a solution, we substitute this expression for y(t) into the differential equation and simplify both sides.

    images

    Since the equation is satisfied for all t ≠ −1, we deduce that y(t) = t + 7 is a solution.

  2. To verify whether y(t) = 3t + 21 is a solution, we substitute this expression for y(t) into the differential equation and simplify both sides.

    images

    Since this equation is not satisfied for all t ≠ −1, we deduce that y(t) = 3t + 21 is not a solution.

  3. To verify whether y(t) = 3t + 9 is a solution, we substitute this expression for y(t) into the differential equation and simplify both sides.

    images

    Since this equation is satisfied for all t ≠ −1, we deduce that y(t) = 3t + 9 is a solution.

Example 2 Verifying an implicit solution to a differential equation

Verify that if y satisfies the relationship

images

then it is a solution to the differential equation

images

Solution From the given equation, we find images:

images

Example 3 From solutions to differential equations

Find a function g(t) such that y(t) = cos t is a solution to

images

on some interval of time.

Solution For y(t) = cos t to be a solution to the given differential equation, we need

images

Therefore, if we chose g(t) = −cot t, then y(t) = cos t is a solution to the differential equation images on a time interval for which both sin t ≠ 0 and cos t ≠ 0.

Separation of variables

As with the case of integration, solving differential equations requires specialized techniques, and there is no guarantee that in general we can find an elementary solution. A special class of differential equations for which we often can find solutions are separable equations–differential equations that can be written in this form:

images

To solve such an equation on an interval of time for which g(y) ≠ 0, we separate the variables to obtain

images

and then integrate both sides separately to obtain

images

The expression we derived for integration by substitution in Section 5.5 implies that the left-hand side can be expressed purely in terms of y (i.e., without reference to t) to obtain the equation

images

which we can then integrate to solve for y in terms of t, as illustrated in the next example.

Example 4 Solving a separable differential equation

Solve

images

Solution In this case g(y) = y and f(t) = −t. Hence, separating variables and integrating yields

images

Solving for y yields y = ±images for any nonnegative C and for −images < t < images.

Notice the treatment of constants in the solution to Example 4. Because all constants can be combined into a single constant, it is customary not to write C = 2(C1C2), but rather to simply replace all the arbitrary constants in the problem by a single arbitrary constant after the last integral is found.

Example 5 Finding and plotting solutions

  1. Solve the differential equation

    images

  2. Find and plot a solution of this equation that satisfies y(0) = 1.

Solution

  1. First observe that y = 0 is a solution, but it is not the solution that passes through the point y(0) = 1. To find this latter solution, we define g(y) = y−2 and f(t) = t and use the separation of variables technique to obtain the integrals.

    images

    To check our work, we can substitute this solution into the differential equation y′(t) = t[y(t)]2 to ensure that both sides are the same.

    images

  2. To satisfy 1 = y(0), we need

    images

    Thus,

    images

    Figure 6.10 Solution to images = ty2 on (−images, images) with y 0 = 1

    images

    The solution is plotted in Figure 6.10 on the interval (−images, images).

Sometimes separation of variables leads to integrals we cannot compute or leads to expressions for which y is only implicitly defined.

Example 6 Implicitly defined solutions

Consider

images

  1. Use separation of variables to solve for y implicitly in terms of t. Use technology to graph this solution.
  2. Find an implicit solution of this equation that satisfies y(−1) = π. Use technology to graph this particular solution.

Solution

  1. In this case, g(y) = y + sin y and f(t) = 2t. Hence, separating variables and integrating yields

    images

    images

    Figure 6.11 Plots of solutions y(t) to the equation images (the horizontal axis y = 0 must be excluded)

    We use technology to plot the solutions as implicitly defined functions yielding the family of solutions shown in Figure 6.11.

  2. To find the constant C from our answer in part a, we substitute y = π and t = −1 into our implicit solution.

    images

    images

    Figure 6.12 Plots of the solutions y(t) to images with y(−1) = π(in blue) and y(−1) = −π(in red).

    Thus, the particular solution is

    images

    and this solution is shown graphically (using technology) in Figure 6.12. Notice that we get two curves in our plot. Only one of them (the blue curve) corresponds to the desired solution. The other curve (in red) corresponds to another solution satisfying the initial condition y(−1) = −π.

Super-exponential and logistic population growth

In Section 1.4 we modeled human population growth in the United States during the nineteenth century with an exponential growth model. Although this model worked reasonably well, the human population on the entire globe appears to be growing faster than exponentially. In the next example, we develop a model of this super-exponential growth.

Example 7 Super-exponential growth and a doomsday prediction

Using data on human abundance across the globe from 1000 AD to 2010 AD, we can model human population growth by a differential equation of the form

images

where N is population size in billions, t is years after 0 AD, b is estimated to be 1.144, and a is estimated to be 0.0035. Assuming that there were 159 million people on Earth in 0 AD, solve for N(t) and estimate in what year the human population will “blow up.”

Solution We want to solve

images

where N(0) = 0.159. Separating the variables and integrating gives

images

Before solving for N, we can solve for C by setting N = 0.159 and t = 0, which yields

images

Solving for N we get

images

Plotting the solution N(t) of our differential equation gives a remarkably good fit to the data, as shown in Figure 6.13.

images

Figure 6.13 Human population growth data from 1000 AD until 2010 AD (as red points) plotted against the solution of the differential equation images 0.0035N2.144 with N(0) = 0.159.

There is a discontinuity of N = (8.2 − 0.004t)−0.874 when 8.2 − 0.004t = 0, equivalently t = 2050. Hence, our model predicts that the population will become infinitely large in 2050 AD. Since no planet can sustain an infinite number of individuals, one can view this solution as a prediction that at the current rate of population growth a doomsday will occur sometime prior to the year 2050.

We know that populations cannot increase to arbitrarily large sizes. The logistic equation, which we introduced in Section 6.1, accounts for limits on population growth. Recall, the logistic equation is

images

where N is the population abundance, r is the intrinsic rate of growth, and K is the carrying capacity. For many data sets, we were able to estimate the parameters r and K. In Example 4 of Section 6.1, we estimated r ≈ 0.53 and K ≈ 660 for the yeast data set of Carlson. Using the method of separation of variables, we can find an analytic solution to this equation and see how well it describes the yeast data of Carlson.

Example 8 Logistic growth of Carlson's yeast data

Find a solution to

images

and compare the solution to the Carlson yeast data set from Example 4 in Section 1.

Solution In this case g(N) = images and f(t) = 0.53. Hence, separating variables and integrating yields

images

To find C corresponding to the solution that passes through the point t = 0 and N = 9.6, we solve

images

To solve for N as a function of t, we use the fact that |N| = N and |660 − N| = 660 − N whenever 0 < N < 660. Hence,

images

images

Figure 6.14 Solution of logistic equation plotted against the Carlson yeast data

A plot of the solution against the Carlson yeast data is shown in Figure 6.14 and illustrates a very good fit.

Note that if N(0) had exceeded 660 in Example 8, the equation to solve for N(t) would be ln N − ln(N − 660) = 0.53t + C, rather than ln N − ln (660 − N) = 0.53t + C as was done above.

PROBLEM SET 6.2

Level 1 DRILL PROBLEMS

Verify in Problems 1 to 8 that if y satisfies the prescribed relationship with t

images

Determine whether the functions given in Problems 9 to 12 are solutions of

images

images

Determine whether the functions given in Problems 13 to 16 are solutions of

images

Solve the differential equations in Problems 17 to 28.

images

Find the solutions to Problems 29 to 36.

images

37. Create a differential equation of the form

images

such that y(t) = et is a solution.

38. Create a differential equation of the form

images

such that y(t) = sin t is a solution.

Level 2 APPLIED AND THEORY PROBLEMS

39. Doomsday prediction. In 1960, three electrical engineers at the University of Illinois published a paper in Science titled “Doomsday.” Based on world population growth data from 1000 AD to 1960 AD, the engineers found that population growth was faster than proportional to the population size. Using the data, they modeled the growth of the population as

images

where P is the population size in billions and t is centuries after 1000 AD. Solve this differential equation and sketch the solution. What year is doomsday according to their model? Compare to the prediction found in Example 7.*

40. The logistic equation did a remarkable job in describing the number of new cases of AIDS in the United States from 1980 until the early 1990s, as seen in Figure 6.15. (see the website http://www.nlreg.com/aids.htm). Let y(t) denote the number of new cases t years after 1980.

Nonlinear regression techniques used to fit the data resulted in the equation

images

  1. Find the solution to this differential equation.
  2. Plot this solution. What happens as t → ∞?
  3. Check the Web to see how this compares with the incidence of AIDS in the United States today. What do you conclude and how do you explain the discrepancy? (There is no right or wrong answer to this part.)

images

Figure 6.15 New cases of AIDS in the United States Permission of Phill Sherrod, http://www.nlreg.com and http://www.dtreg.com

41. A model for tumor growth is the Gompertz function that is a solution to the differential equation

images

where y is the weight of tumor in milligrams, t is measured in days, a is a constant, and K is the limiting size of the tumor. Assume that a = 0.5 and K = 100.

  1. Find a solution to this differential equation that satisfies y(0) = 1 mg.
  2. Plot this solution.

42. The 1984 U.S. census recorded a population of 15,757,000 Hispanics; in 1990, the size was 16,098,000. Assuming that the rate of population growth is proportional to the population, predict the population of Hispanics in the United States in the year 2000. Use the Web to find the actual Hispanic population in 2000. How does your prediction compare with the actual number? What do you think can account for any differences?

43. Consider a chemical reaction involving two reactants, A and B, that form a product C. Let [A], [B], and [C] denote the concentrations of A, B, and C. If a molecule of A encounters molecules of B at a rate proportional to their concentration, then the law of mass action states that

images

where k is a positive constant. If the initial concentration of A is a, the initial concentration of B is b, and we set y = [C], then [A] = a − [C], [B] = b − [C], and

images

Assume that a = b. Find and plot the solution to this differential equation satisfying y(0) = 0.

44. Populations may exhibit seasonal growth in response to seasonal fluctuations in resource availability. A simple model accounting for seasonal fluctuations in the abundance N of a population is

images

where R is the average per capita growth rate and t is measured in years.

  1. Assume R = 0 and find a solution to this differential equation that satisfies N(0) = N0. What can you say about N(t) as t → ∞?
  2. Assume R = 1 (more generally R > 0) and find a solution to this differential equation that satisfies N(0) = N0. What can you say about N(t) as t → ∞?
  3. Assume R = −1 (more generally R < 0) and find a solution to this differential equation that satisfies N(0) = N0. What can you say about N(t) as t → ∞?

6.3 Linear Models in Biology

An important class of models is described by the linear differential equation

images

where the constants a and c are model parameters that have specific physical or biological interpretations. For example, in Section 6.1 we saw how models with a = 0 and c = r were used to describe exponential population growth (c = R) and radioactive decay (c = −λ). In this section, we discuss more applications where the constant coefficient a is nonzero.

Mixing models

Mixing models formulated on the premise that the density of individuals or concentration of molecules, which are generically characterized in terms of a number of objects per unit area or volume, form a homogeneous pool. The flow of objects into the pool is controlled by an external constant rate, while the flow of objects out of the pool is in proportion to the density of objects in the pool. This latter assumption implies that the greater the density of objects in the pool, the faster the total flow of objects out of the pool.

Mixing Model

Let y(t) represent the density of objects in a pool at time t. If objects flow into this pool at a constant total rate a > 0 and out of this pool at a rate by(t) > 0, that is, at a constant per capita clearance rate b > 0, then the density of objects in the pool over time is governed by the equation

images

Our first application of mixing models is to obtain a better understanding of within-host pathogen dynamics of HIV. Human immunodeficiency virus-type 1 (HIV-1) has many puzzling quantitative features. For instance, most persons with HIV undergo a ten-year period during which concentration of the virus in plasma is very low. It is only after this quiescent period that a person experiences the onset of AIDS. The reason for this quiescent period is unknown, and it had been presumed that during this period the virus was relatively inactive. Using models, as described in the next example, Perelson and colleagues quantified viral levels in the blood of infected individuals during this quiescent period.*

Example 1 Modeling HIV levels

In building a model of HIV levels in infected hosts, Perelson and colleagues used the variable V(t) to represent the concentration of viral particles in a host's blood plasma (Figure 6.16). They assumed that HIV viral particles infused into the blood, from production sites in lymphatic tissue, at a constant rate a > 0 and were cleared from the blood at a rate bV. From these assumptions, they obtained the mixing model

images

where t is measured in days. Both a and b are unknown, positive constants.

images

Figure 6.16 An Illustration of HIV in a host's bloodstream

  1. Data showed that after a host received a potent antiviral drug, viral concentration in the blood fell exponentially. Assuming that the drug rapidly fell to zero, Perelson and colleagues estimated the half-life of the viral particles to be 0.2 days. Use this information to estimate the clearance rate constant, b.
  2. Perelson and colleagues estimated that prior to administration of the drug, the mean plasma viral level was 2.16 · 105 viral particles per milliliter (ppmL). Assume that before the administration of the drug the system was at equilibrium (i.e., V is such that dV/dt = 0). Using the estimate of b from part a, estimate the rate a of production of viral particles from the lymphatic tissue prior to administration of the drug.

Solution

  1. To estimate the clearance rate of all the viral particles currently in the blood of an individual, we set the production parameter a to zero, which yields

    images

    We solved this equation previously and found the general solution V(t) = V(0)ebt where V(0) is the initial viral load. Since the half-life is 0.2 days, we know that images when t = 0.2. Thus,

    images

    which gives b = 5 ln 2 ≈ 3.47.

  2. If the viral density in a person's blood is at equilibrium prior to administration of the drug, then images = 0 and we can solve for a.

    images

    Hence, we estimate about 75 × 104 viral particles per milliliter per day. The typical individual has approximately 5.6 liters of blood, which means during the quiescent phase

    images

    viral particles are being created per day. Thus, this dormant phase still exhibits “the raging fire of active HIV replication.” Consequently, Perelson and colleagues suggested that “early and aggressive therapeutic intervention is necessary if a marked clinical impact is to be achieved.”

Hospital patients often receive drugs by intravenous infusion. For drugs to be administered effectively and safely, correct infusion rates must be determined. Differential equation models are a basic tool used by doctors to determine these infusion rates. These models are known as pharmacokinetics or biopharmaceutics models. (Check out http://www.boomer.org/c/p1/index.html for a whole course on this topic.)

Example 2 Determining an infusion rate

A patient who has asthma is given a continuous infusion of theophylline to relax and open the air passages in his lungs. The desired steady-state level of theophylline in the patient's bloodstream is 15 mg/L. The average half-life of theophylline is about four hours, and the patient has 5.6 L of blood in his body.

  1. Find the necessary infusion rate.
  2. Determine how long it takes for the concentration of theophylline to be 10 mg/L.

Solution

  1. First, we write a differential equation model. Let y(t) be the amount (milligrams) of theophylline in the blood plasma at time t, in hours. Let a denote the rate (milligrams per hour) at which theophylline enters the bloodstream via infusion. Let b denote the clearance rate constant of the theophylline. Then,

    images

    To determine b, we use the fact that the half-life of theophylline is about four hours. What this means is that in the absence of the infusion (i.e., when a = 0), half of the theophylline leaves the blood plasma in four hours. Solving y′(t) = −by(t) yields y(t) = y(0)ebt. Since images, we can solve for b as follows:

    images

    To find a, we want the equilibrium (i.e., the y value for which images = 0) to hold at y = 15 mg/L × 5.6 L = 84 mg:

    images

    The desired infusion rate is approximately 14.3 mg/h.

  2. To determine how long it takes to reach a concentration of 10 mg/L, we need to solve the differential equation subject to the initial condition y(0) = 0 (i.e., initially, there is no drug in the patient). First, using the separation of variables method, the solution for any value of a and b is

    images

    Now we solve for C3 corresponding to the solution that passes through y = 0 at t = 0. This implies C3e0 = a/b or simply C3 = a/b. Since a = 14.28 and b = 0.17, images = 84, and the particular solution is

    images

    Finally, since a concentration of 10 mg/L corresponds to having 10 × 5.6 mg = 56 mg in the blood, we need to solve

    images

    Thus, it will take about 6.5 hours for the concentration of theophylline to reach 10 mg/L.

In the solution to Example 2b, we found the general solution to the mixing model, which we restate for future reference.

General Solution of the Mixing Model

The general solution of the differential equation

images

is given by

images

where C is an arbitrary constant.

In Example 1, we determined the clearance rate constant when the half-life of the viral particles in the patient's blood was known. In Example 2, we determined the infusion rate necessary to maintain a desired concentration of drug in the patient's blood. Next, in Example 3, we determine the clearance rate constant when the half-life of the viral particles in the patient's blood is not known.

Example 3 Determining a clearance rate constant

Consider a patient who is receiving a drug intravenously at a rate of 10 mg/h. An hour later, the concentration of drug in the patient's body is 1 mg/L. Assume that the patient has 5 L of blood and that the drug is lost at a rate proportional to the amount of drug in the body; find the clearance rate constant of the drug. Finally, determine the limiting concentration of drug in the patient's body.

Solution Let y denote the amount of drug in the patient's body. Then, because the patient has 5 L of blood, y/5 is the concentration of drug per liter in the patient's body. The rate of change of y is given by

images

where 10 is the infusion rate and b > 0 is the clearance rate constant. Either redoing our separation of variables work or directly using the general solution of the mixing model, we get

images

To solve for C, we use the initial condition y(0) = 0, which gives 0 = y(0) = imagesC. Therefore, C = images and we have

images

To find the clearance rate constant b, we can use the fact that

images

Now, we need to solve

images

We cannot solve for b analytically in this equation. Hence, we use technology to find b ≈ 1.6 is a solution to this equation.

To find the limiting concentration of the drug in the patient's body, we must find the limit of y(t) as t approaches ∞.

images

The limiting quantity is 6.25 mg in 5 L of blood, or 1.25 mg/L.

Another application of the mixing model is the following analysis of the concentration of pollution in a lake.

Example 4 Lake pollution

A well-mixed lake with a constant volume of 100 km3 is fed by rivers and tributaries at a rate of 48 km3/year. Factories are dumping polluted water, as depicted in Figure 6.17, into the lake at a rate of 2 km3/year. Environmental studies have shown that after mixing, if the percentage of water in the lake from polluted sources exceeds 2%, then the water becomes a hazardous environment for fish.

images

Figure 6.17 A pipe dumping polluted water into a lake

  1. Does this lake ever reach the 2% level of polluted water mixed in with fresh water? If so, when?
  2. How much would the polluted water input into the lake have to be decreased to reduce the long-term mixture of water from polluted sources to 2%?

Solution

  1. Let y (units in cubic kilometers) denote the total amount of polluted water mixed into the lake. Assume that y(0) = 0. The rate polluted water enters the lake is 2 km3/year. Finding the rate at which pollutants are leaving is more difficult. Since the lake volume is assumed to be constant, the rate at which the well-mixed water leaves the lake is equal to the rate at which the fresh and polluted water enter the lake; that is, 48 + 2 = 50 km3/year. The proportion of polluted water in the lake is y/100. Hence, the rate at which polluted waters leave the lake is

    images

    Thus, our initial value problem is

    images

    with initial value y(0) = 0. Using the general solution for the mixing model with a = 2 and b = 1/2, we get

    images

    The initial condition y(0) = 0 implies that C = 4, and

    images

    Since images y(t) = 4, the eventual amount of polluted water in the well-mixed lake is 4 km3, which is 4% of the lake's 100 km3 volume. Thus, the lake will reach the hazardous level. To find the time at which it reaches the 2% hazardous level, we solve

    images

    This is about 1 year, 5 months.

  2. To ensure that the polluted water never exceeds 2% of the total, we reformulate the model as

    images

    where p is the rate polluted water is dumped into the lake, and the initial value is y(0) = 0. Polluted water levels in the lake will approach the equilibrium level given by

    images

    Hence, if the flow of polluted water is reduced to 1 km3/year, then the long-term (i.e., equilibrium) proportion of polluted water in the lake is 2%.

Newton's cooling law and forensic medicine

In forensic medicine, determining the time of death of a victim can be critical in convicting the murderer. Mathematics can aid this process by using Newton's law of cooling. This law states that the rate at which the temperature of a body changes is proportional to the difference between the body's temperature T and the ambient temperature A. Mathematically, this statement corresponds to the following differential equation:

images

where k a positive constant proportionate to the thermal conductivity of the body. A large k means the body readily conducts heat and quickly adjusts to the ambient temperature. A small k means the body is well insulated and slowly adjusts to the ambient temperature. Notice that this differential equation is also a mixing model where y = T, a = kA, and b = k. Hence, the general solution of this cooling model is

images

where C is an arbitrary constant.

Example 5 Put on your detective caps

On a dark and stormy night, Sherlock Holmes and Dr. Watson were called to investigate the shocking murder of Jacob Marley. The main suspects of this crime were three people who would benefit from his death. First, there was Marley's business partner, Ebenezer Scrooge, who was having strong disagreements with Marley about how to run the business. Scrooge spent the evening alone working late at his office. His household staff confirmed that he arrived home at 9 P.M. and remained home for the rest of the night. Second, there was Marley's wife, Claudia, who was having an affair with another man. Claudia stood to inherit Marley's fortune. Claudia was at the theater from 8 P.M. to 9:30 P.M., as verified by several people at the theater. Finally, there was Marley's client, Sam Wise Gange, whom Marley swindled out of a large sum of money. Sam was at a local pub until 11:00 P.M., which was verified by several people. Marley's body was found in an alley at 1:30 A.M. The alley temperature was a nippy 55 degrees and the body temperature was 87°F. One hour later, the body temperature had cooled to 85°F. Given this information, determine who has a good alibi.

Solution Let T denote the temperature of the body. By Newton's law of cooling

images

Using the general solution of this differential equation, we get

images

To determine C and k, let us identify t = 0 with 1:30 A.M. Since at this time the body temperature was 87°F, it follows that

images

Thus,

images

To find k, we use the information that the body temperature was 85°F an hour after 1:30 A.M., which implies

images

Finally, to determine the time of death, we need to solve backwards in time to the point where the body temperature was a normal 98.6°F, that is,

images

Thus, Marley died approximately four hours and forty-five minutes before the body was found, that is, at approximately 8:45 P.M. Thus, Claudia and Sam have alibis for the murder, while Scrooge does not.

Organismal growth

The study of growth involves determining body size as a function of age. Various measurements of body size exist, including weight, length, and girth. A famous equation, the von Bertalanffy growth equation, which describes growth of an organism, can be derived from first principles using scaling laws. To derive this equation, consider a cubical critter with length L as illustrated in Figure 6.18.

images

Figure 6.18 Cubical critter

The surface area of this critter is 6L2 and the volume is L3. If we assume that length is measured in centimeters and that the critter is mostly made of water, which has a density of 1 g/cm3, then the critter's mass M is L3 grams. If a critter ingests food at a rate proportional to its surface area and respires at a rate proportional to its mass, then

images

where a and b are positive proportionality constants. Since M = kL3, where the value of k depends on units of measurement and so can be set to 1 without affecting the form of the solution (the values of a and b would be adjusted according to the selected units), we obtain

images

Combining the previous two equations yields

images

Defining k = b/3 and L = a/b yields the two-parameter von Bertalanffy growth equation

images

The next example clarifies why biologists use the notation L for one of the parameters.

Example 6 von Bertalanffy growth equation

Find a general solution to the von Bertalanffy growth equation

images

with initial value L(t0) = 0; that is, the organism was “born” at time t0.

Solution This differential equation is just another mixing model with y = L, a = kL, and b = k. Hence, the general solution is

images

Next, we use the initial condition L(t0) = 0 to find C

images

Thus,

images

images

Figure 6.19 von Bertalanffy growth equation.

As shown in Figure 6.19. This time t0 is sometimes thought of as the theoretical time of conception, but it is only a meaningful concept if the same growth equation applies at all stages of development (which is not generally a reasonable assumption).

PROBLEM SET 6.3

Level 1 DRILL PROBLEMS

In Example 1 we modeled HIV levels using the data of Perelson and colleagues. In that example we assumed that the half-life of the viral particles was 0.2 days (or 4.8 hours) and that the mean plasma viral level was 2.16 · 105 viral particles per milliliter (ppmL). Estimate the clearance rate constant b for the half-life given in Problems 1 to 6 and then estimate the daily rate of production of HIV particles for the specified mean plasma viral level.

1. 2.4 h; 1.89 · 105 viral ppmL

2. 3 h; 2.15 · 105 viral ppmL

3. 4 h; 2.25 · 105 viral ppmL

4. 5 h; 2.35 · 105 viral ppmL

5. 6 h; 3.15 · 105 viral ppmL

6. 7.2 h; 2.75 · 105 viral ppmL

Using the information from Example 2, determine the length of time it takes for the concentration of theophylline to be the quantity given in Problems 7 to 12.

7. 5 mg/L

8. 7 mg/L

9. 12 mg/L

10. 14 mg/L

11. 14.5 mg/L

12. 14.99 mg/L

Find the amount of drug in a patient's body as a function of time t, given the infusion rate and the concentration of the drug one hour later, as given in Problems 13 to 18. Assume the patient has 5 L of blood.

13. 10 mg/h; 1.6 mg/L

14. 12 mg/h; 1 mg/L

15. 12 mg/h; 2 mg/L

16. 20 mg/h; 1 mg/h

17. 20 mg/h; 2 mg/h

18. 20 mg/h; 3 mg/h

Rework Example 4 using all of the given information and changing only the lake size and outflow as shown in Problems 19 to 24.

19. Lake size of 50 km3 and outflow of 23 km3/year

20. Lake size of 100 km3 and outflow of 62 km3/year

21. Lake size of 100 km3 and outflow of 23 km3/year

22. Lake size of 120 km3 and outflow of 10 km3/year

23. Lake size of 50 km3 and outflow of 18 km3/year

24. Lake size of 80 km3 and outflow of 10 km3/year

Level 2 APPLIED AND THEORY PROBLEMS

In Problems 25 to 28, set up an appropriate model to answer the given question. These problems use a special case of the linear model in which the relative rate of change remains constant.

25. In 1990 the gross domestic product (GDP) of the United States was $5,464 billion. Suppose the growth rate from 1989 to 1990 was 5.08%. Predict the GDP in 2003. Check your answer by finding the actual 2003 GDP.

26. In 1980 the gross domestic product (GDP) of the United States in constant 1972 dollars was $1,481 billion. Suppose the growth rate from 1980 to 1984 was 2.5% per year. Predict the GDP in 2003. Check your answer by finding the actual 2003 GDP.

27. According to the Department of Health and Human Services, the annual growth rate in the number of divorces per year in 1990 in the United States was 4.7% and there were 1,175,000 divorces that year. How many divorces will there be in 2004 if the annual growth rate in the number of divorces per year remains constant?

28. According to the Department of Health and Human Services, the annual growth rate in the number of marriages per year in 1990 in the United States was 9.8% and there were 2,448,000 marriages that year. How many marriages will there be in 2004 if the annual growth rate in the number of marriages per year is constant?

29. The rate at which a drug is absorbed into the blood system is given by

images

where x(t) is the concentration of the drug in the bloodstream at time t. What does x(t) approach in the long run (that is, as t → ∞)? At what time is x(t) equal to half this limiting value? Assume x(0) = 0.

30. Calculate the infusion rate in milligrams per hour required to maintain a long-term drug concentration of 50 mg/L (i.e., the rate of change of drug in the body equals zero when the concentration is 50 mg/L). Assume that the half-life of the drug is 3.2 hours and that the patient has 5 L of blood.

31. Calculate the infusion rate in milligrams per hour required to maintain a desired drug concentration of 2 mg/L. Assume the patient has 5.6 L of blood and the half-life of the drug is 2.7 h.

32. Calculate the infusion rate required to achieve a desired drug concentration of 2 mg/L in 1 h. Assume the elimination rate constant of the drug is 5 mg/L per hour and the patient has 6 L of blood.

33. Calculate the infusion rate required to achieve a desired drug concentration of 12 mg/L in 20 min. Assume the clearance rate constant is 2 mg/L per hour and the patient has 5 L of blood.

34. A drug is given at an infusion rate of 50 mg/h. The drug concentration value determined at 3 h after the start of the infusion is 8 mg/L. Assuming the patient has 5 L of blood, estimate the half-life of this drug.

35. A drug is given at an infusion rate of 250 mg/h. The drug concentration determined at 4 h after the start of the infusion is 50 mg/L. Assuming the patient has 5.5 L of blood, estimate the elimination rate constant of this drug.

36. A lake with a constant volume of 10,000 m3 is initially clean and pristine. Water flows into the lake from two streams, Babbling Brook and Raging Rapids, at rates of 250 m3 per day and 750 m3 per day, respectively. At time t = 0, road salt from a nearby road contaminates Babbling Brook with concentration of 2 kg/m3. Find an equation that describes the amount of salt in the lake for all t ≥ 0 and find the limiting amount of salt in the lake.

37. After one hydrodynamic experiment, a tank contains 300 L of a dye solution with a dye concentration of 2 g/L. To prepare for the next experiment, the tank is to be rinsed with water flowing in at a rate of 2 L/min, with the well-stirred solution flowing out at the same rate. Write an equation that describes the amount of dye in the container. Be sure to identify variables and their units.

38. At midnight the coroner was called to the scene of the brutal murder of Casper Cooly. The coroner arrived and noted that the air temperature was 70°F and Cooly's body temperature was 85°F. At 2 A.M., she noted that the body had cooled to 76°F. The police arrested Cooly's business partner Tatum Twit and charged her with the murder. She has an eyewitness who said she left the theater at 11:00 P.M. Does her alibi help?

39. A cup of coffee at a cafe is served at 95°C and left on the counter. The cafe is air-conditioned with an ambient temperature of 20°C. After five minutes, the coffee's temperature is 45°C. Determine how long before the coffee loses its taste quality; that is, it cools down to the temperature of 22°C.

40. In 1986 the Chernobyl nuclear disaster in the Soviet Union contaminated the atmosphere. The buildup of radioactive material in the atmosphere satisfies the differential equation

images

where M = mass of radioactive material in the atmosphere after time (in years); k is the rate at which the radioactive material is introduced into the atmosphere; r is the annual decay rate of the radioactive material. Find the solution, M(t), of this differential equation in terms of k and r.

41. The von Bertalanffy curve was used to examine growth patterns in both body length and mass of female and male polar bears (Ursus maritimus) captured live near Svalbard, Norway (see Figure 6.20). A longer growth period in males resulted in pronounced sexual dimorphism in both body length and mass. Males were 1.16 times longer and 2.10 times heavier than females. For females, L = 194 cm, k = 0.75/year, and t0 = −0.27 is the theoretical age at which the polar bear would have no length (L0 = 0). For males, L = 225 cm, k = 0.537/year, and t0 = −0.395 is the theoretical age at which the polar bear would have no length. Use the von Bertalanffy curve to determine at what age males and females achieve half of their limiting size.

images

Figure 6.20 The von Bertalanffy curve fitted to age and body length: data for female (blue dots and curve) and male (red dots and curve) polar bears.

Data Source: A. E. Derocher and images Wiig, “Postnatal Growth in Body Length and Mass of Polar Bears (Ursus maritimus) at Svalbard,” J. Zool., Lond. 256 (2002): 343–349.

42. In Chapter 5 Section 8, we introduced survival-renewal equations to describe how populations (e.g. money, organisms) change when individuals arrive at rate r(t) and survive with probability s(t) for t units of time. If initially the population size is y(0), then the survival-renewal equation is

images

If s(t) = eat, then verify that y(t) solves the linear differential equation:

images

6.4 Slope Fields and Euler's Method

Not all equations are separable, and many separable equations do not lead to explicit solutions. Furthermore, even when you find a solution, it may be so complex that it is nearly impossible to interpret. To address these issues, we discuss a qualitative method, slope fields, and a numerical method, Euler's method, for studying solutions of differential equations.

Slope fields

Consider a differential equation of the general form

images

where f(t, y) denotes an expression involving t and y. Since a solution y(t) to this differential equation satisfies y′(t) = f(t, y(t)), it follows that the slope of all solutions at time t are given by the right-hand side f(t, y(t)) of the differential equation. Equivalently, f(t, y(t)) is the slope of the tangent line of y(t) at time t. A qualitative way to investigate the behavior of solutions to y′(t) = f(t, y) is to sketch its slope field, a figure in the ty plane with infinitesimal line segments of slope f(t, y) at (t, y).

Consider, for example, sketching the slope field for images. Since for t = 1 the slope is images = 1, we draw short line segments at t = 1, each with slope 1, for different y values, as shown in Figure 6.21a. For t = −3, the slope is −1/3 and we draw short line segments at t = −3, each with slope −1/3, also shown in Figure 6.21a. If we continue to plot these slope points for different values of t, we obtain many little slope lines. The resulting graph, shown in Figure 6.21b, is the slope field for the equation y′(t) = 1/t.

We can also examine the relationship between the slope field for y′(t) = 1/t and its solutions y = ln |t| + C. If we choose particular values for C, say C = 0, C = −ln 2, or C = 2, and draw these particular solutions as shown in Figure 6.21c, we see that these solutions are tangent to the slope field.

images

Figure 6.21 Solution of the differential equation y′ = images using a slope field

The slope field for y′(t) = 1/t was relatively straightforward to sketch, but such sketches can be more challenging for differential equations with a more complicated right-hand side. In the next example, we illustrate how to obtain a qualitative understanding using nullclines.

Nullclines

The set of points (t, y) for which the equation

images

holds are called the nullcline(s). For most models, these nullclines correspond to a finite number of curves. In general, however, they are a closed subset of the ty-plane.

Example 1 Sketching solutions using a slope field

Consider a drug that continuously infuses into a patient at a periodic rate. One possible differential equation modeling such a scenario is

images

where y is the amount of drug (in milligrams) and t is the time in units of hours. Notice that this is just a mixing problem (see Example 2 of Section 3.4) where the input is now the time-dependent infusion rate 10 + 10 sin t mg/h and the elimination rate constant is 1 mg/h. Sketch the slope field for this differential equation, and sketch the particular solution that satisfies y(0) = 0.

Solution To sketch the slope field by hand, it often suffices to determine

images

and

images

In this example, the nullclines satisfies

images

which implies

images

Since everywhere along the curve y = 10 + 10 sin t the slope images is 0, we draw small line segments with slope 0 along this curve.

Since images < 0 whenever y > 10 + 10 sin t, we draw line segments with negative slopes above the nullcline such that the slopes of these line segments get closer to 0 as the line segments get closer to the curve y = 10 + 10 sin t. Similarly, since images > 0 whenever y < 10 + 10 sin t, we draw line segments with positive slopes below the nullcline. This recipe yields a sketch similar to what is shown in Figure 6.22a.

images

Figure 6.22 Slope field and a particular solution

To sketch the solution satisfying y(0) = 0, we sketch a curve starting at t = 0, y = 0 that remains tangent to the slope field, which leads to a sketch similar to Figure 6.22b. This qualitative analysis correctly suggests that this solution eventually exhibits well-defined oscillations. In fact, the conclusion can be verified by solving this differential equation using the solution to Problem 42 in Problem Set 6.3.

Slope fields can provide qualitative insights when solutions are only implicitly defined or are very complicated. In the next example, the equations are separable and an explicit solution can be found. The solution, however, requires solving for the roots of a cubic equation, yielding complicated expressions that are hard to interpret. The behavior of the solution, however, is quite easy to infer using slope fields. The example is inspired by the work of Warder Clyde Allee, an American ecologist who argued that the per capita growth rate of populations may be negative when population densities are low, despite being positive at higher densities. Reasons for this so-called Allee effect can be due to cooperative hunting or the difficulty of finding mates at low population densities.

Example 2 The Allee effect

Consider the population model

images

where N is the population density, t is time, r > 0 and 0 < A < K. Use the parameters r = 1, K = 200, and A = 50.

  1. Sketch the per capita growth rate function.
  2. Sketch the slope field.
  3. Sketch and compare solutions satisfying N(0) = 49 and N(0) = 55.

Solution

  1. The per capita growth rate function for the specified parameters is

    images

    and is illustrated on the left.

    images

    Consistent with the Allee effect, the per capita growth rate is negative at low densities yet positive at intermediate densities. At high densities it is negative again due to competitive effects.

  2. To sketch the slope field, we first solve for the nullclines. This corresponds to the set of points in the tN-plane for which

    images

    Hence, the nullclines are given by the lines N = 0, N = 50, and N = 200 in the tN-plane. Along the lines, we sketch horizontal line segments.

    images

    between the lines N = 0 and N = 50 and above the line N = 200, we sketch line segments with negative slope. Moreover, the slope of these line segments gets closer to zero as the line segments get closer to N = 0, N = 50, or N = 200.

    For 50 < N < 200, we have images > 0 (shaded blue). Hence, between the lines N = 50 and N = 200, we draw line segments with positive slope. Moreover, the slope of these line segments gets closer to zero as the line segments get closer to N = 50 or N = 200. This work yields a sketch similar to Figure 6.23a.

    images

    Figure 6.23 Slope field with two particular solutions

  3. To sketch a solution satisfying N(0) = 49, we sketch a curve passing through the point t = 0, N = 49 that remains tangent to the slope field (i.e., “go with the flow”). This curve is shown in Figure 6.23b; it starts at (0, 49) and becomes asymptotic to the t axis.

    Similarly, we sketch a solution satisfying N(0) = 55, which is shown in Figure 6.23b starting at (0, 55) and becoming asymptotic to the line N = 200.

    These solutions suggest that whenever 0 < N(0) < A, the population declines to extinction. Whenever N(0) > A, the population converges to K.

Any model of the form y′ = f(y) is called autonomous because the associated slope field is independent of time (i.e., the function f(y) does not explicitly depend on time). We discuss these equations in much greater detail in the next two sections. The counterparts to these autonomous models are purely time dependent equations images = f(t)—that is, the slope field does not depend at all on the variable y. In this case, a solution y(t) is an antiderivative of f(t).

Example 3 Purely time-dependent slope field

Consider

images

Find all solutions to this differential equation; then sketch several members of the family of curves representing the solutions. Finally, use technology to compare the slope field with this family of solutions.

Solution We begin by separating the variables and integrating.

images

If we sketch the solutions for a variety of C values, we obtain a family of downward facing parabolas as illustrated in Figure 6.24a.

images

Figure 6.24 Family of solutions compared with slope-field solution

Now, use technology to graph the slope field for y′(t) = −t. As expected, the slope lines are tangents to the downward facing parabolas, as illustrated in Figure 6.24b.

Using slope fields, we sometimes can quickly answer questions about the long-term behavior of solutions to a differential equation.

Example 4 Lake pollution revisited

In Example 4 of Section 6.3 we showed that the amount of polluted water y(t) (in cubic kilometers) at time t (years) from factories dumping effluent at a rate of 2km3/year into a well-mixed lake of constant volume 100 km3, fed by rivers and tributaries at a rate of 50 km3/year, is given by the equation

images

Sketch a slope field for this equation and use it to find the limiting values as t → ∞.

Solution We used technology to draw the slope field in Figure 6.25. Note that the nullcline is

images

and that the slopes above and below this nullcline are negative and positive, respectively.

images

Figure 6.25 Slope field and solutions to the equation images

Sketches of several solutions on this slope field are shown in Figure 6.25. Hence, as we had previously shown analytically, the volume of polluted water in the well-mixed lake approaches a limiting value of y = 4km3 (or equivalently, an asymptotic concentration of 4% polluted water).

Euler's method

Sometimes it is not possible to solve for the solution of a differential equation analytically; nevertheless, we want more than a qualitative sense of the solution. Such situations require numerical methods. The simplest numerical method is Euler's method, which roughly corresponds to sliding short linear segments along the slope field. More precisely, Euler's method for the differential equation

images

involves choosing a step size h > 0 and making the approximation

images

Rearranging this expression gives

images

Therefore, if we are given y at time t, we can approximate y at time t + h. Applying this approximation iteratively yields an approximation to the solution of the differential equation.

Euler's Method

Consider the equation

images

To estimate the solution of this equation, follow these steps:

Step 1: Choose a step size h > 0 and define t1 = t0 + h, t2 = t0 + 2h,...

Step 2: Estimate y(t1) by y1 where

images

In other words, approximate y(t1) using the tangent line to y(t) at t0, as illustrated in Figure 6.26a.

images

Figure 6.26

Step 3: Estimate y(t2) by y2 where

images

In other words, approximate y(t2) following the slope field at (t1, y1).

Step 4: Repeat this process and at the i-th step estimate y(ti) by yi where

images

as illustrated in Figure 6.26b.

Euler's method is illustrated in the following example.

Example 5 Euler's method

Use Euler's method with h = 0.1 to estimate the solution of the initial value problem

images

over the interval [0, 0.5].

Solution To use Euler's method for this example, note:

images

For h = 0.1, the Euler approximation (correct to four decimal places) is:

images

images

Figure 6.27 Comparison of an analytically derived solution (blue) and the approximate solution due to Euler's method (red).

These points can be plotted to approximate the solution, as shown by the red line in Figure 6.27–a reasonable approximation of the analytically derived solution shown in blue.

Example 6 Comparing Euler's method for two time steps h

Use Euler's method to approximate the solution to

images

on the interval [0, 2] with y(0) = 0.

  1. Solve by hand, with h = 0.5.
  2. Solve using technology, with h = 0.1.

Solution

  1. We have f(t, y) = sin πty with h = 0.5, t0 = 0, and y0 = 0. Thus, by Euler's method we obtain

    images

    To visualize the solution, we plot these points in the ty-plane, and connect them with line segments, as shown in the left panel of Figure 6.28.

    images

    Figure 6.28 Euler approximation of images = sin πty for h = 0.5 (left panel) and h = 0.01 (right panel)

  2. We use technology to graph a slope field, along with the solution using Euler's method for h = 0.1, as shown in the right panel of Figure 6.28.

The one thing that is obvious in comparing the solutions graphed in the left and right panels of Figure 6.28 is that approximate solutions with larger h values can be crude. They may even go horribly wrong, as in the next example. However, as the step size decreases, the time it takes to complete the calculation increases. Thus it is important, as with any numerical scheme, to have error bounds to determine how small h needs to be to get an answer to within a desired level of accuracy. The ultimate trade-off involves accuracy versus time to complete the calculation. Although we do not discuss error bounds for differential equations in this book, you can learn about them in any introductory numerical analysis course.

Example 7 Effect of the choice of h

Consider the logistic equation

images

which has this solution (using separation of variables):

images

Compare the plots on [0, 5] of the numerical solution and actual solution for the given values of h.

  1. h = 0.1
  2. h = 0.08
  3. h = 0.05

Solution

  1. Using Euler's method for h = 0.1 on [0, 5] yields 51 values for t and y. Plotting these values in the ty-plane yields the red curve shown in Figure 6.29a. The actual solution is shown in blue. As you can see, the numerical solution acts quite wildly.
  2. Using Euler's method for h = 0.08 on [0, 5] yields 63 values for t and y. Comparing these values (red) with the actual solution (blue). As we can see, Figure 6.29b reveals that the numerical solution is not a good approximation, even though it is not quite as wild as that shown in part a.
  3. Using Euler's method for h = 0.05 on [0, 5] yields a 101 values for t and y. Comparing these values (red) with the actual solution (blue). Note that Figure 6.29c reveals that the numerical and actual solutions are almost identical.

images

Figure 6.29 Euler approximations for solution of images

The moral of the previous example is that with the Euler method, as with other numerical methods for solving differential equations, one must be careful in choosing the appropriate step size h.

PROBLEM SET 6.4

Level 1 DRILL PROBLEMS

Sketch at least three particular solutions for each of the slope fields shown in Problems 1 to 6.

images

images

images

Sketch a solution satisfying the specified initial conditions shown over the slope field in Problems 7 to 10.

7. y(0) = 0.3

images

8. y(0) = 2

images

9. y(6) = 0

images

10. y(8) = 2

images

11. Match the following four equations with the four slope fields.

images

images

12. Match the following four equations with the four slope fields.

images

images

Sketch the slope fields and sketch a few solutions for the differential equations given in Problems 13 to 18.

images

Sketch the slope fields and the solution passing through the specified point for the differential equations given in Problems 19 to 24.

images

Estimate a solution for Problems 25 to 28 using Euler's method. For each of these problems, a slope field is given with an actual solution. Superimpose the segments from Euler's method on the given slope field and assess how well your solution approximates the actual solution as drawn.

images

images

Use Euler's method to approximate the solution to images = f(t, y) and sketch the approximate solution in Problems 29 to 32 over the specified interval.

29. Over the interval 0 ≤ t ≤ 2 with f(t, y) = (4 − y)(y + 2), y(0) = 0.1, h = 0.5

30. Over the interval 0 ≤ t ≤ 1 with f(t, y) = yt, y(0) = 2, h = 0.2

31. Over the interval 0 ≤ t ≤ 3.5 with f(t, y) = sin πt − 2y, y(0) = 0, h = 0.5

32. Over the interval −1 ≤ t ≤ 0 with f(t, y) = (4 − y)(y + 2), y(−1) = 0, h = 0.2

33. Consider the differential equation

images

  1. Verify that y(t) = ln t is a solution to this differential equation satisfying y(1) = 0.
  2. Use Euler's method to approximate y(2) = ln 2 with h = 0.5.

34. Consider the differential equation

images

  1. Verify that y(t) = et is a solution to this differential equation satisfying y(0) = 1.
  2. Use Euler's method to approximate e with h = 0.2.

Level 2 APPLIED AND THEORY PROBLEMS

35. A patient receives a continuous drug infusion of 100 mg/h. The half-life of the drug is two hours.

  1. Write a differential equation for the amount of drug in the body. (Hint: Review Example 2 in Section 6.3.)
  2. Sketch the slope field for this differential equation.
  3. Determine the limiting amount of the drug in the patient's body.

36. A patient receives a continuous drug infusion of 50 mg/h. The half-life of the drug is one hour.

  1. Write a differential equation for the amount of drug in the body. (Hint: Review Example 2 in Section 6.3.)
  2. Sketch the slope field for this differential equation.
  3. Determine the limiting amount of the drug in the patient's body.

37. A population subject to seasonal fluctuations can be described by the logistic equation with an oscillating carrying capacity. Consider, for example,

images

Although it is difficult to solve this differential equation, it is easy to obtain a qualitative understanding.

  1. Sketch a slope field over the region 0 ≤ t ≤ 5 and 0 ≤ P ≤ 200.
  2. Sketch solutions that satisfy P(0) = 0, P(0) = 10, and P(0) = 200.
  3. Use technology to obtain a better rendition of the slope field and solutions.
  4. Comment on your solutions and compare to your work using different methods.

38. The velocity v(t) of a skydiver is governed by the equation

images

where m is the mass of the skydiver, g is gravitational acceleration, and k is a dampening constant (i.e., accounts for air friction).

  1. Sketch the slope field for this equation assuming that m = 70 kg, g = 9.8m/s2, and k= 110 kg/s.
  2. Using the slope field, determine the value of images v(t) for the solution v(t) satisfying v(0) = 0. Note that this limiting value is known as the terminal velocity.

39. An autocatalytic chemical reaction involves two molecules, A and B. Let a denote the concentration of A and assume that the concentration b of B remains constant throughout the experiment (e.g., B is added to the mixture in such a way to keep b constant). If A combines with a molecule of B to form two molecules of A and in a backward reaction, two molecules of A form a molecule of A and B, then

images

where k1 and k2 are positive rate constants.

  1. Sketch the slope field for this equation for the case k1 = 1, b = 1, k2 = 0.5.
  2. For the cases a(0) = 0.2 and a(0) = 3, sketch in the solutions and determine the value of images a(t).

40. A population, in the absence of harvesting, exhibits the following growth

images

where N is abundance and t is time in years.

  1. Write an equation that corresponds to harvesting the population at a rate of 0.5% per day.
  2. Sketch the slope field for the differential equation you found in part a; by sketching solutions, describe how the fate of the population depends on its initial abundance.

6.5 Phase Lines and Classifying Equilibria

Now we focus on autonomous (independent of time) differential equations:

images

The slope fields for autonomous differential equations are time independent. Since the slope field at any point in time contains all the information about the slope field, an infinite amount of redundancy exists. In this section we remove this redundancy using phase lines, and we discuss classifying equilibria, the y values for which f(y) = 0. With this new qualitative approach to studying autonomous differential equations, we examine evolutionary games and voltage drops across cell membranes.

Phase lines

In the previous section, we sketched slope fields by determining where the slope is zero (nullcline), where it is positive, and where it is negative. In this section, we consider a phase-line diagram that collapses the two-dimensional slope field to the y axis without losing any information about the qualitative behavior of solutions to images = f(y) as illustrated Figure 6.30.

images

Figure 6.30 Illustration of how the slope field for the logistic equation images = y(1 − y), can be collapsed on the y axis by removing (or projecting down) the time axis t.

Phase Lines

To draw a phase line for images = f(y), follow these steps:

Step 1. Draw a vertical line corresponding to the y axis.

Step 2. Draw solid circles on this line corresponding to the equilibria of images = f(y), that is, y values where f(y) = 0.

Step 3. Draw an upward arrow on intervals where f(y) > 0. On these intervals, solutions of the differential equation are increasing.

Step 4. Draw a downward arrow on intervals where f(y) < 0. On these intervals, the solutions of the differential equation are decreasing.

In the next example we consider competition between two clonal populations of the same species. A clonal population consists of genetically identical individuals who only have mothers (reproducing asexually) and are all descendants of the same female. Clonal populations include many kinds of plants, fungi, and bacteria, and some insects, fish, amphibians, and reptiles such as the whip-tail lizard shown in Figure 6.31.

images

Figure 6.31 Certain species of whiptail lizards are clonally reproducing; that is, all individuals are female and all progeny are identical genetic copies of their mothers, except for changes due to mutations.

Example 1 Competition of clonal genotypes

Consider two clonally reproducing lines of the same species that exhibit two genotypes denoted by a and A. Suppose the growth rate of each line satisfies an exponential growth model, with the per capita growth rates of lines a and A being ra and rA, respectively. Further, suppose these two clonal lines are growing together in the same geographic area, and let y denote the proportion of genotype a in this population. The solution to Problem 39 in Problem Set 6.5 shows that the variable y satisfies the equation

images

  1. Draw the phase line for this equation when ra > rA.
  2. Draw the phase line for this equation when ra < rA.
  3. Discuss why your answers to parts a and b make sense.

Solution

  1. We begin by drawing the y axis. The equilibria are determined by the solutions of

    images

    Since the equilibria are y = 0 and y = 1, we draw solid circles on the y axis at these y values. Since ra > rA, we have images > 0 for 0 < y < 1 and we draw an upward arrow on this interval. Since images < 0 for y > 1 and y < 0, we draw downward arrows on these intervals. This results in the phase line illustrated in Figure 6.32a.

  2. Again begin by drawing the y axis. The equilibria are determined, as before, by the solutions of

    images

    Since the equilibria are y = 0 and y = 1, we draw solid circles on the y axis at these y values. Since ra < rA, we have images < 0 for 0 < y < 1 and we draw a down-ward arrow on this interval. Since images > 0 for y > 1 and y < 0, we draw upward arrows on these intervals. This results in the phase line illustrated in Figure 6.32b.

    images

    Figure 6.32 Phase lines for images = (rarA)y(1 − y)

  3. If the per capita growth rate of genotype a is greater than the per capita growth rate of genotype A, then we would expect genotype a to become more and more prevalent in the population. Hence, provided that y > 0 initially, y approaches 1 as seen in the phase line for part a. Conversely, if the per capita growth rate of genotype a is less than the per capita growth rate of genotype A, then we would expect a to become less and less prevalent in population. Hence, y should approach 0, as seen in the phase line for part b.

In the previous example, we found phase lines from an equation, but sometimes we have a graph of f(y) and not an equation. The next example shows how to find phase lines in such a case.

Example 2 From graphs to phase lines to solutions

Let the graph of f(y) defined on the domain (−∞, ∞) be as shown in Figure 6.33.

  1. Draw a phase line for images = f(y).
  2. Sketch solutions for this differential equation that satisfy y(0) = −1.1, y(0) = 1.1, and y(0) = 0.9.

Solution

  1. Since the graph of f(y) intersects the y axis at the points −2, −1, 1, and 2, these y values are the equilibria of y′ = f(y). We draw solid circles at these points of the phase line. Since f(y) > 0 on the intervals (−∞, −2) and (1, 2), we draw upward arrows on these intervals, as shown in Figure 6.34. For all the other intervals, (−2, −1), (−1, 1), and (2, ∞), we draw downward arrows.

    images

    Figure 6.33 Graph of images = f(y)

    images

    Figure 6.34 Phase line (left) and solutions (right) to the differential equation depicted in Figure 6.33

  2. According to the phase line, a solution initiated at y = −1.1 initially decreases slowly (as it is near the equilibrium y = −1), decreases more rapidly, and asymptotes at the equilibrium y = −2. A solution initiated at y = 1.1 initially increases slowly, increases more rapidly, and asymptotes at the equilibrium y = 2. A solution initiated at y = 0.9, initially decreases slowly, decreases more rapidly, and asymptotes at the equilibrium y = −1. These solutions are shown in Figure 6.34.

The equation y = (rarA)y(1 − y) in Example 1 has a special name in the context of evolutionary game theory. It is called the replicator equation. Evolutionary games were formalized into a coherent mathematical theory in the late 1970s by the theoretical evolutionary biologist John Maynard Smith (1920–2004). (For more information about one of the world's greatest evolutionary biologists, see the images in Problem Set 6.5.) Perhaps the best known of his games is the Hawk–Dove game, which describes under what conditions aggressive versus non-aggressive behaviors persist in populations.

In general, for any two inherited contrasting strategies, the growth rates ra and rA for genotype a (e.g., doves) and genotype A (e.g., hawks), respectively, in the replicator equation are constructed from a two-by-two table. This table is known as the payoff matrix; it tells how much payoff (benefit if positive, cost if negative) an individual gets after a pairwise interaction with another individual. The payoffs when a meets a are denoted by Paa. Similarly, PaA, PAa, and PAA denote the payoffs when a meets A, A meets a, and A meets A, respectively. This information is summarized in Table 6.3.

Table 6.3 Payoff Matrix that specifies the payoff to individuals in the rows interacting with individuals in the columns.

images

If the payoff affects an individual's reproductive rate, we determine the per capita (i.e., proportional) growth rate of a genotype. We find the expected payoff by calculating the product of the chance of meeting an individual playing a particular strategy and the corresponding payoff.

For genotype a, this expected payoff is

images

and for genotype A the expected payoff is

images

Substituting these expressions for ra and rA into y′ = y(1 − y)(rarA), we get the two-strategy replicator equations.

Two-Strategy Replicator Equation

The replicator equation describing the dynamics of the proportions y and (1 − y) of the population of genotype a and A, with payoff matrix Pij, (i, j = a and A) is

images

John Maynard Smith and George Price used replicator equations to understand why conflicts between individuals within species rarely escalate to fights to the death. For instance, in their classic 1973 paper “The Logic of Animal Conflict,” Nature 246: 15–18, they wrote with regard to mule deer (Figure 6.35):

“In mule deer (Odocoileus hemionus ) the bucks fight furiously but harmlessly by crashing or pushing antlers against antlers, while they refrain from attacking when an opponent turns away, exposing the unprotected side of its body.”

images

Figure 6.35 Two male mule deer engaging in a contest for a mate. Such furious conflicts rarely lead to either individual being seriously hurt.

To model animal conflicts, they considered a population of individuals competing for a limiting resource such as mates, food, or shelter. To win this resource, individuals engage in pairwise contests and play one of two strategies, hawk or dove. Individuals playing the hawk strategy constantly escalate the intensity of the contest until either they get the resource or they get injured. Individuals playing the dove strategy leave the contest whenever their opponent escalates the conflict. This game is illustrated in the next two examples.

Example 3 The Hawk-Dove replicator equation

Suppose a hawk gets a payoff of V > 0 every time it meets a dove and the dove gets 0. Every time two doves meet they share the payoff V, while if two hawks meet they escalate the contest until one gets the net payoff V and the other pays a cost C > 0. What are the payoff matrix entries for this contest and the replicator equation that describes the frequency of doves in the population?

Solution Let a represent doves and A represent hawks. In this game, the payoffs are as follows:

images

If we substitute these values in the two-strategy replicator equation we obtain

images

Example 4 Dynamics of a Hawk-Dove game

Consider a Hawk-Dove game with the payoff V = 2 and the cost C = 3. Sketch the phase line and discuss the evolutionary implications.

Solution When V = 2 and C = 3, we obtain from the previous example the specific replicator equation

images

The equilibria solutions are values for which dy/dt = 0:

images

For 0 < y < 1/3, images > 0. To see this, choose a value in the interval, say y = images, and calculate

images

For 1/3 < y < 1, images < 0. To see this, choose a representative value, say y = images, and calculate

images

Finally, for y > images > 0. To see this, choose some representative value, say y = 2, and calculate

images

The phase line is shown in Figure 6.36. The phase line implies that if initially hawks and doves are present, then the population approaches an equilibrium consisting of one-third doves and two-thirds hawks. This approach to equilibrium is illustrated in Figure 6.36.

images

Figure 6.36 Phase line (left) and solutions (right) to the equation images = y(1 − y)(1 − 3y)/2

The equilibrium value y = 1/3 in Example 4 supports both the hawk and dove strategies in the proportion of one individual playing the dove strategy for every two individuals playing the hawk strategy. Such an equilibrium is called a polymorphic strategy, as opposed to a monomorphic strategy where all individuals play either hawk or dove. In Example 4, we can understand the growth rates of hawks and doves at low frequencies as follows. Imagine the population consists mainly of doves and only a few hawks. In this case, individuals are most likely to have a contest with a dove. For a dove this means getting a payoff of V/2 = 2 × (1/2) = 1. For a hawk this means getting a payoff of V = 2. Therefore, hawk numbers will grow at twice the rate of doves. Alternatively, consider a population consisting mostly of hawks and only a few doves. In this case, individuals are most likely to encounter a hawk. For a hawk this means an average payoff of (VC)/2 = (2 − 3)/2 = −1/2. For a dove this means a payoff of 0. Therefore, hawk numbers will decline until a balance is reached at the polymorphic equilibrium.

Classifying equilibria

When a system starts at an equilibrium, it remains there for all time. In the real world, however, biological systems are constantly subject to environmental perturbations. Thus, if a system starting at equilibrium is slightly perturbed from equilibrium, we need to ask if it tends to return to the equilibrium or not. When the system tends to return to the equilibrium, we call the equilibrium stable. Otherwise, we call it unstable. Precise definitions follow; see Figure 6.37 for a graphical representation.

images

Figure 6.37 Graphical representation of the classification of equilibria

Classification of Equilibria

An equilibrium y* of the equation

images

that is a solution satisfying

images

can be classified as follows:

Stable: f(y) > 0 for all y < y* near y* and f(y) < 0 for all y > y* near y*. Solutions initiated near the equilibrium tend toward the equilibrium in forward time (i.e., as t → ∞).

Unstable: f(y) < 0 for all y < y* near y* and f(y) > 0 for all y > y* near y*. Solutions initiated near the equilibrium tend toward the equilibrium in backward time (i.e., as t → −∞).

Semistable: Either f(y) < 0 for all yy* near y* or f(y) > 0 for all yy* near y*. Solutions initiated near one side (respectively, other side) of the equilibrium tend toward the equilibrium in backward (respectively, forward) time.

Example 5 Classifying equilibria

Classify the equilibria for images = f(y) where the graph of f(y) is the graph given in Figure 6.33 in Example 2, which we repeat here for convenience.

images

Solution Previously, we sketched the phase line for images = f(y) and found four equilibria: y = −2, y = +2, y = −1, y = +1. From the phase line sketch in Example 2, Figure 6.34, we classify the equilibria as follows: y = −2 and y = 2 are stable, y = 1 is unstable, and y = −1 is semistable.

The voltage V across the membrane of a typical cell is maintained by voltage-gated (i.e., controlled) protein channels embedded in the cell membrane. These channels (see Figure 6.38) regulate the flow of positively charged potassium ions and negatively charged organic molecules out of the cell, and negatively charged chlorine ions and positively charged sodium ions into the cell.

images

Figure 6.38 An illustration of a voltage-gated channel depicting the electrical field gradients and ions that flow across cell membranes

Example 6 Membrane potential

If a cell membrane is perturbed from its resting potential V0 by a small input current (e.g., coming from another neuron), it will return to its resting potential. However, if this perturbing current is sufficiently large to cause V(t) to drop below a critical threshold level Vc, then the sodium ions flow across the membrane until the voltage stabilizes at a new depolarized equilibrium level Vd. Show that model

images

exhibits these characteristics by finding and classifying its equilibria for the values V0 = −70 millivolts (mV), Vc = −30 mV, Vd = 55 mV, and k = 1.

Solution For the constants in question, the right-hand side of the equation is

images

This function is cubic in the variable V with roots at V = −70, −30, and 55. The graph of this cubic is given by the graph shown in Figure 6.39.

Since f(V) > 0 for V less than but close to −70 and f(V) < 0 for V less than but close to −70, V0 = −70 mV is stable. Similarly, Vd = 55 mV is stable. In contrast, since f(V) < 0 for V less than but close to −30 and f(V) > 0 for V more than but close to −30, Vc = −30 mV is unstable. Hence, the membrane returns to its resting potential, −70 mV, whenever perturbations are small. However, if there is a sufficiently large perturbation to cause the membrane potential to be above −30 mV, then the membrane approaches the depolarized state of 55 mV. Moreover, small perturbations from this depolarized state will not return the membrane to its resting potential. Only a sufficiently large perturbation will.

images

Figure 6.39 Graph of F(V) = −(V + 70) × (V + 30)(V − 55)

Linearization

An analytical approach to classifying equilibria involves linearizing the right-hand side of the differential equation

images

about its equilibria. To this end, suppose y* is an equilibrium of this equation and we evaluate the derivative of f(y) at y* to obtain a = f′(y*). A linear approximation to f(y) for y near y* is given by

images

Hence,

images

for values of y near y*. As you are asked to show in Problem 37 of Problem Set 6.5, the solution to

images

satisfying y(0) = y0 is

images

We can use this solution as a first-order approximation for the solution to images = f(y) satisfying y(0) = y0. This approximation y(t) = (y0y*)eat + y* is reasonable provided y(t) remains near y*. If a < 0, then this approximation suggests that images y(t) = y* whenever y0 is sufficiently close to y*. Alternatively, if a > 0, then this approximation suggests that imagesy(t) = y* whenever y0 is sufficiently close to y*. These observations can be made mathematically precise and can be used to prove the following theorem.

Theorem 6.1 Stability of a first-order autonomous differential equation

Suppose f(y) is differentiable at y = y* and y = y* is an equilibrium of the equation

images

Then y* is

Stable if f′(y*) < 0,

Unstable if f′(y*) > 0, and

Indeterminate if f′(y*) = 0 (no conclusion is possible without looking at higher-order derivatives)

Beyond the stated result, we can use the approximation

images

to gauge at what “speed” solutions move toward or away from y*. The more positive a is, the more rapidly solutions move away from y*. The more negative a is, the more rapidly solutions approach y*.

Example 7 Population resilience

Consider two populations whose dynamics are described by

images

  1. Find the equilibria and use linearization to classify their stability properties.
  2. Describe in what ways the populations are similar and dissimilar.

Solution

  1. For both populations, the equilibria are given by 0 and 10,000. For the first model, we have f(N) = N(1 − N/10,000) and

    images

    Evaluating this derivative at the equilibria, we find:

    images

    For the second model, we have g(P) = 0.5 P images and can find:

    images

  2. The populations are similar in that both populations have equilibria at 0 and 10,000, which are unstable and stable, respectively. Hence, both populations tend to approach the equilibrium value of 10,000.

    The populations differ in that population P (the second model) tends to grow less rapidly at low densities; that is, g′(0) = images < f′(0) = 1. Moreover, if the populations are at the equilibrium of 10,000, the P population recovers less rapidly from a perturbation; that is, g′(10,000) = −images > f′(10,000) = −1.

When one population recovers more rapidly from environmental perturbations than another population (as with P versus N in Example 7), it is said to be more resilient.

Example 8 Hawk-Dove game revisited

Consider the Hawk-Dove game

images

where y is the frequency of doves in the population.

  1. Use linearization to classify each of the equilibria.
  2. Use your work from part a to determine whether the hawks increase more rapidly at low frequencies or the doves increase more rapidly at low frequencies.

Solution

  1. Let

    images

    As we have seen, the equilibria are y = 0, y = 1, and y = 1/3. To linearize, we need this derivative:

    images

    Evaluated at y = 0, we obtain

    images

    Hence, the equilibrium y = 0 is unstable.

    Since

    images

    the equilibrium y = 1 is unstable.

    Since

    images

    the equilibrium y = images is stable.

  2. Since f′(1) = 1 > f′(0) = images, we see that hawks increase more rapidly at low frequency than doves do.

Example 9 Linearization of membrane voltage model

Consider the membrane potential model from Example 6:

images

Assume that V0 < Vc < Vd and k > 0.

  1. Use linearization to classify the equilibria.
  2. Discuss the resilience of the two stable equilibria.

Solution

  1. Define

    images

    The roots of the function f(V) are the equilibria V* = V0, Vc, and Vd. Two applications of the product rule imply

    images

    Thus, at V = V0, we have

    images

    Since V0 < Vc < Vd, f′(V0) < 0 which implies V0 is stable.

    At V = Vc, we have

    images

    Since V0 < Vc < Vd, f′(Vc) > 0 which implies Vc is unstable.

    At V = Vd, we have

    images

    Since V0 < Vc < Vd, f′(Vd) < 0 which implies V = Vd is stable.

  2. The resting state V0 is more resilient than the depolarized state Vd whenever

    images

    Hence, provided the difference between the resting potential and the critical potential is greater than the difference between the depolarized potential and the critical potential, the resting state is more resilient.

PROBLEM SET 6.5

Level 1 DRILL PROBLEMS

Draw phase lines, classify the equilibria, and sketch a solution satisfying the specified initial value for the equations in Problems 1 to 10.

images

images

images

Draw a phase line for images = f(y) for the graphs shown in Problems 11 to 14. Sketch the requested solutions.

11. y(0) = −1.1, y(0) = 1.1, y(0) = 0.9

images

12. y(0) = −0.1, y(0) = 0.9, y(0) = 1.1

images

13. y(0) = −2, y(0) = 1, y(0) = 1.5

images

14. y(0) = −0.1, y(0) = 1.9, y(0) = 3.9

images

Linearize about the equilibrium in Problems 15 to 20 and classify stability properties.

images

Sketch the phase line and classify the equilibria for the Hawk-Dove game with the values V and C given in Problems 21 to 24.

21. V = 2, C = 2

22. V = 4, C = 2

23. V = 3, C = 2

24. V = 2, C = 4

Sketch the phase line and classify the equilibria for the replication equations with the indicated payoffs in Problems 25 to 28.

25. Paa = 2, PaA = 1, PAa = 1, and PAA = 2

26. Paa = 1, PaA = 2, PAa = 3, and PAA = 4

27. Paa = −1, PaA = 2, PAa = 1, and PAA = −1

28. Paa = 2, PaA = −1, PAa = −1, and PAA = 3

Level 2 APPLIED AND THEORY PROBLEMS

29. Consider a pack of wolves that can either jointly hunt a stag or individually on their own hunt hares. Suppose during the course of a stag hunt, a hare comes along and an individual wolf considers either remaining with the pack to hunt the stag or going off on its own to hunt the hare. Suppose the payoffs for these two strategies (remain with the pack or go off and hunt the hare) are given by the following payoff matrix in the context of an evolutionary game:

images

  1. Find the replicator equation, assuming that in any one hunt only one wolf in the pack can make the decision to go or not go after the hare.
  2. Sketch the phase line and classify the equilibria.
  3. Discuss how the outcome of the evolutionary game depends on the initial strategy composition of the population.

30. Consider two scenarios based on Problem 29:

  • In a population of stag hunters, a few individuals decide to hunt hares.
  • In a population of hare hunters, a few individuals decide to hunt stags.

Use linearization to determine in which of these scenarios the “defecting” individuals are more rapidly excluded.

31. Evolution of cooperation, part 1. Consider a population with two strategies, cooperate and defect. Individuals that cooperate provide a benefit B to their opponents and pay a cost C for providing this benefit. Defectors provide no benefits to their opponents and pay no cost. Under these assumptions, we get the following payoff matrix:

images

  1. Write a replicator equation for this payoff matrix.
  2. Assuming B > 0 and C > 0, sketch the phase line for the replicator equation.
  3. Discuss the implications of your phase line.

32. Evolution of cooperation, part 2. In Problem 31, cooperation could not evolve. However, cooperation occurs in natural populations. In this problem, we investigate how individuals that interact frequently and respond to the strategy of their opponents can promote the evolution of cooperation. Imagine that each time two opponents meet they interact an average n times. Individuals can play one of two strategies: defect always or tit-for-tat, in which case an individual initially cooperates but switches to defecting if its opponent defected.

  1. If each time individuals interact the payoffs are as in Problem 31, then discuss why the payoff matrix should be this:

    images

  2. Write a replicator equation for this game.
  3. Assume B = 3 and C = 2. Sketch phase lines for n = 2, 3, 4.
  4. Discuss the implications for the evolution of cooperation.

33. To account for the effect of a generalist predator (with a type II functional response) on a population, ecologists often write differential equations of the form

images

where N is the population abundance and t is time (in years). The first term of the equation corresponds to logistic growth and the second term corresponds to saturating predation.

  1. Sketch the phase line for this system.
  2. Discuss how the fate of the population depends on its initial abundance.

34. Construct the phase line for the model

images

and demonstrate that this equation belongs to the class of membrane voltage models presented in Example 6.

35. Use a phase line diagram to discuss the behavior of the membrane voltage models presented in Example 6 with constants k = 3, V0 = −65 mV, Vc = 40 mV, and Vd = 40 mV. Does this membrane have the property that it is able to switch between two states when perturbed by a current?

36. images

images

John Maynard Smith, or JMS as he was almost always known, was a professor at the University of Sussex (United Kingdom), and one of the world's great evolutionary biologists. JMS introduced mathematical modeling from game theory into the study of mathematical biology and completely revolutionized the way that biologists think about behavioral evolution. Jonathan Weiner in his book titled A Conversation with John Maynard Smith describes JMS as a classical geneticist and leading theorist in evolutionary biology. Weiner also relates how JMS applied game theory to explain jousting matches among many species, including sticklebacks, sea lions, stag beetles, and stags.

JMS received many prestigious awards. For this images, research and write a few words about each of the following awards received by JMS, or, in the case of the last one, established in his honor.

  1. Balzan Prize
  2. Crafoord Prize
  3. Kyoto Prize
  4. John Maynard Smith Prize

37. Verify that the solution to

images

satisfying y(0) = y0 is given by

images

38. Show that the linearization theorem is inconclusive when the derivative equals zero at the equilibrium by considering the stability of the equilibria to these equations:

images

39. Consider a population of clonally reproducing individuals consisting of two genotypes, a and A, with per capita growth rates, ra and rA, respectively. If Na and NA denote the densities of genotypes a and A, then

images

Also, let y = images be the fraction of individuals in the population that are genotype a. Show that y satisfies

images

40. In the Hawk-Dove replicator equation

images

if the value V > 0 is specified, then find the range of values of C (in terms of V) that will ensure a polymorphism exists (i.e., find conditions that ensure the existence of an equilibrium 0 < y* < 1 that is stable).

41. Production of pigments or other protein products of a cell may depend on the activation of a gene. Suppose a gene is autocatalytic and produces a protein whose presence activates greater production of that protein. Let y denote the amount of the protein (say, micrograms) in the cell. A basic model for the rate of this self-activation as a function of y is

images

where a represents the maximal rate of protein production, k > 0 is a “half saturation” constant, and b ≥ 1 corresponds to the number of protein molecules required to active the gene. On the other hand, proteins in the cell are likely to degrade at a rate proportional to y, say cy. Putting these two components together, we get the following differential equation model of the protein concentration dynamics:

images

  1. Verify that images A(y) = a and A(k) = a/2.
  2. Verify that y = 0 is an equilibrium for this model and determine under what conditions it is stable.

42. Consider the model of an autocatalytic gene in Problem 41 with b = 1, k > 0, a > 0, and c > 0.

  1. Sketch the phase line for this model when ck > a.
  2. Sketch the phase line for this model when ck < a.

43. Sketch the phase line for the autocatalytic gene model in Problem 41 with b = 2, k = 1, and c = 1.

  1. Sketch the phase for this model when a = 1.
  2. Sketch the phase for this model when a = 2.
  3. Sketch the phase for this model when a = 2.5.
  4. Discuss what you found.

6.6 Bifurcations

Biological systems can exhibit a range of dynamic behaviors that change abruptly or gradually in response to external perturbations. The term bifurcation is used in the context of difference or differential equation models to denote a change in the qualitative behavior of the solutions that occurs as a parameter in the model is varied–that is, as one of the constants in the model changes its value. In this section, we introduce bifurcation theory. We will use the notation

images

to represent an expression in y and a where y is a model variable and a is a parameter. The goal is to understand how the behavior of the solutions of this differential equation depends on a. More precisely, we will study how the phase line varies with the parameter a.

We will examine bifurcation theory in populations subjected to harvesting, protein dynamics associated with an autocatalytic gene, and the firing rates of neural populations.

Sudden population disappearances

Example 1 Harvesting queen conchs

Consider a population of queen conchs in the Bahamas whose dynamics are given by

images

where t is time in years, y is number of conchs, and a is the constant annual harvesting rate.

  1. Draw phase lines for a = 0, a = 21,000, and a = 30,000.
  2. Discuss the biological implications of these phase lines.
  3. Discuss how the number of equilibria depends on the harvesting rate a.

Solution The growth rate function f(y) = 10y(1 − y/10,000) is plotted in Figure 6.40 (blue curve) along with the values of a (broken horizontal lines) and associated equilibria (vertical dotted lines).

images

Figure 6.40 Annual growth rate of conchs is plotted as a function of population density (blue curve) subject to harvesting at three levels: 0, a = 21,000 (broken green line), a = 25,000 (broken brown lines) and a = 30,000 (broken red line). The vertical dotted lines indicate the associated equilibria, as discussed in the text.

  1. Consider a = 0. The equilibria are given by the solutions of

    images

    Solving this equation yields the equilibria y = 0 and y = 10,000. Since images > 0 for 0 < y < 10,000 and images < 0 for the other intervals, we obtain the phase line on the left in Figure 6.41. Now consider a = 21,000. The equilibria are given by solutions to

    images

    images

    Figure 6.41 Phase lines for the density (y) of conchs for the three harvesting levels a, as labeled, inserted into the conch harvesting equation. The direction arrows indicate which points are stable (i.e., approached) and which are unstable.

    which yields y = 3,000 and y = 7,000 (where the green dotted line intersects the blue curve in Figure 6.40). Since images > 0 for 3,000 < y < 7,000 and images < 0 elsewhere, we get a phase line as shown in the center of Figure 6.41. Finally, consider a = 30,000 (the red dotted line in Figure 6.40). In this case there are no equilibria because

    images

    is negative for all y ≥ 0 and we get a phase line as shown on the right-hand side of Figure 6.41.

  2. The phase lines in Figure 6.41 show that as a increases, the number of equilibria goes from two to zero. In particular, at sufficiently high harvesting rates, the population is unable to persist at an equilibrium.
  3. To determine how the equilibria depend on the harvesting rate a, we need to solve

    images

    for y. Using the quadratic formula

    images

    we obtain two equilibria provided that

    images

    which occurs if and only if a < 25,000. If a = 25,000 (brown dotted line in Figure 6.40), then we get only one equilibrium given by y = 5,000. Finally, if a > 25,000, then 2,500− images is negative and there are no equilibria. Therefore, a change in the number of equilibria occurs at a = 25,000.

Example 1 illustrates that the phase line of images = f(y, a) can vary substantially as we vary the parameter a. Moreover, it shows that at certain parameter values (i.e., a = 25,000 in Example 1) there is a qualitative change in the phase line. These values are important enough to have their own name: bifurcation values. A bifurcation value is defined as the value of a parameter in an equation where either the number of equilibrium solutions changes or the stability of these equilibria undergo a transition from stable to unstable. A simple way to graphically summarize how the behavior of the system depends on a is to create a bifurcation diagram.

Bifurcation Diagram

A bifurcation diagram summarizes the behavior of the differential equation images = f(y, a) in the ay plane and is created as follows:

Step 1. Draw the a axis (horizontal) and the y axis (vertical).

Step 2. Sketch the set of equilibria in the ay plane, that is, the set of points (a, y) that satisfy 0 = images = f(y, a).

Step 3. Determine in which regions of the ay plane, images is positive or negative.

Step 4. For a collection of a values, draw a phase line. In particular, draw phase lines at bifurcation values of a and at values of a that lie between bifurcation values.

Example 2 Sudden disappearances of queen conchs

Sketch a bifurcation diagram for Example 1:

images

with a ≥ 0 and y ≥ 0. Discuss the implications for harvesting queen conchs.

Solution We begin by solving

images

for a and graphing a = 10y(1 − y/10,000) in the ay plane. The graph is a parabola as shown in Figure 6.42a.

images

Figure 6.42 The curve of equilibria and bifurcation diagram for the differential equation images

Choosing a point inside the parabola, say (0, 5,000), we obtain images = 10 × 5,000(1 − 1/2) = 25,000 > 0. Hence, images > 0 inside of the parabola. Choosing a, point outside of the parabola, say (10, 0), we obtain images = −10 < 0. Hence images < 0 outside of the parabola.

Next, we sketch phase lines for several a values, say a = 5,000, a = 15,000, a = 25,000, and a = 30,000. For each of these values of a, we draw a vertical line. Where the line intersects the parabola, we draw a solid circle (in red in Fig. 6.42b) as this corresponds to points where images = 0. Where the line lies inside the parabola, we draw an upward arrow. Where the line lies outside the parabola, we draw a downward arrow. The resulting bifurcation diagram is illustrated in Figure 6.42b. Notice that for a = 0, a = 21,000, and a = 30,000, we get the same phase lines as in Example 1.

This bifurcation diagram indicates that for 0 < a < 25,000 there are two equilibria. The lower equilibrium is unstable and the upper equilibrium is stable. When the two equilibria coalesce at a = 25,000, the resulting equilibrium is semistable—that is, solutions starting above the density y = 5,000 decrease to asymptotically approach the equilibrium y = 5,000, while solutions that start below 5,000 also decrease to 0.

It follows that for harvesting rates over the range 0 < a < 25,000, the population can persist provided that its initial population abundance is sufficiently large (e.g., above 5,000 for the case a = 25,000). Moreover, the stable population equilibrium is always greater than 5,000. On the other hand, if the population is harvested at a rate a > 25,000, it will eventually be driven to 0 regardless of the initial abundance y(0). Note that from a modeling point of view, harvesting must necessarily be set to 0 at y(t) = 0, since a population that has 0 individuals can no longer be harvested. From a purely mathematical point of view, though, the equation remains valid for y(t) < 0.

An important implication of the bifurcation diagram in Example 2 is that gradual changes in harvesting can bring about discontinuous changes in population abundance. More specifically, when the harvesting rate is ever so slightly increased beyond the bifurcation value (a = 25,000 in Example 1), the population begins slowly at first, but then more rapidly, to decline from high abundance to extinction. Such population disappearances have been observed in natural populations. Dramatic examples include the precipitous drop of blue pike (Stizostedion vitreum glaucum) from annual catches of 10 million pounds to less than a thousand pounds in the mid-1950s, or the unexpected collapse of the Peruvian anchovy (see Figure 6.43) population in 1973, as illustrated in Figure 6.44, and the sudden reduction of Great Britain's grey partridge (Perdix perdix) population in 1952.

images

Figure 6.43 The Peruvian anchovy (Engraulis ringens) once supported the biggest fishery of all time with a catch of 13.1 million metric tons in 1970.

images

Figure 6.44 Catch data for Peruvian anchovies in the twentieth century

Data Source: Table 3 in: Castillo, S., & Mendo, J. (1987). Estimation of unregistered Peruvian anchoveta (Engraulis ringens) in official catch statistics, 1951–1982. In D. Pauly, & I. Tsukayama (Eds.), The Peruvian anchoveta and its upwelling ecosystem: Three decades of change. ICLARM studies and reviews (Vol. 15, pp. 109–116)

The bifurcation occurring at a = 25,000 in the queen conch example is a saddle node bifurcation, because the transition from two equilibria to zero equilibria is preceded by the appearance of a semistable (or saddle) equilibrium. A more colorful name for this bifurcation is blue sky catastrophe, as two equilibria vanish into the blue sky as a increases past the value 25,000. Another important type of bifurcation is the pitchfork bifurcation illustrated by the next example. A look ahead at Figure 6.45 indicates the source of this name: one equilibrium bifurcates into three to create a pitchfork.

Example 3 Pitchfork bifurcation

Sketch a bifurcation diagram for

images

Solution The equilibria are given by

images

Hence, at an equilibrium, either y = 0 or y2 = a. The sketches of the curves y = 0 for all a and a = y2 for all y in the ay plane yields the blue lines in Figure 6.45. These curves determine four regions in the ay plane: the regions above and below the pitchfork and the upper and lower parabolic wedges of the pitchfork. Using the point (a, y) = (0, 1), we obtain images = −1 < 0. Hence, images < 0 in the region above the pitchfork. Using the point (a, y) = (0, −1), we obtain images = 1 > 0. Hence, images > 0 in the region below the pitchfork. Using the point (a, y) = (2, 1), we obtain images = 1 > 0. Hence, images > 0 in the upper parabolic wedge of the pitchfork. Using the point (a, y) = (2, −1), we obtain images = −1 < 0. Hence, images < 0 in the lower parabolic, wedge of the pitchfork.

images

Figure 6.45 Diagram of the pitchfork bifurcation associated with the differential equation images = yy3 obtained by plotting the equilibria images = 0 (blue lines) and using vertical phase lines to (red lines) illustrate how the dynamics vary with a.

To complete the bifurcation diagram, it suffices to sketch phase lines for a negative a value (i.e., only one equilibrium), a positive a value (i.e., three equilibria), and the bifurcation value a = 0. Drawing vertical lines at these a values, solid circles at the equilibria, upward arrows where images > 0, and downward arrows where images < 0, results in the completed bifurcation diagram illustrated in Figure 6.45.

Biological switches

In order to respond appropriately to changes in their environment, living cells receive environmental signals and respond to these signals in variety of ways, including gene expression and cell activity. Similar to your computer, information processing is carried out by a complex network of switches. Unlike your computer, these switches are not made from electronic parts; rather, they consist of interacting genes and the proteins activated by these genes. Here, we examine two forms of biological switches, an autocatalytic gene introduced in Problem 41 of Section 6.5 and a neural switch for memory.

Example 4 An autocatalytic gene

As discussed in Problem 41 of Section 6.5, production of pigments or other protein products of a cell may depend on the activation of a gene. The gene can be autocatalytic; that is, it produces a protein whose presence activates the production of more protein. If the gene requires two proteins to bind to its receptor region to activate production of the protein, and if y denotes the concentration of the protein, then a simple model of the dynamics of y is given by

images

where a represents the maximal rate of protein production, k > 0 is a “half saturation” constant, and c is the per capita degradation rate of the protein. For k = 1 and c = 1, sketch a bifurcation diagram of the dynamics for a > 0 and y ≥ 0.

Solution Setting the parameters k = 1 and c = 1 leaves us with the following model:

images

To solve for the equilibria as a function of a, we set this differential equation to zero and factor out a y term

images

This equation is satisfied if either y = 0 or

images

Sketching the equilibrium curves y = 0 and a = images + y in the ay plane yields the blue curves in Figure 6.46a. Note that there are two regions in the nonnegative quadrant: the region where a < images + y and the region where a > images + y. At the point (a, y) = (1, 2) in the region a < images + y, we have images = 2(1/5 − 1) < 0. Hence, in this region, images < 0. At the point (a, y) = (4, 1) in the region a > images + y, we have images = 1(4/2 − 1) > 0. Hence, in this region, images > 0.

images

Figure 6.46 Constructing a bifurcation diagram for a model of an autocatalytic gene

Since there is only one equilibrium for a < 2, two equilibria at a = 2, and three equilibria for a > 2, we sketch a phase line for a = 1, 2, and 3. At a = 1, there is only equilibrium at y = 0 and images < 0 for all y > 0. We get the left phase line in Figure 6.46b. At a = 2, there are equilibria at y = 0 and near y = 1. Since images < 0 for all other y, we get the center phase line in Figure 6.46b. At a = 3, there are equilibria at y = 0, near y = images, and near y = 2.5. We have images < 0 for y between the first two equilibria, images > 0 for y between the latter two equilibria, and images < 0 for larger y values. Hence, we get the right phase line in Figure 6.46b.

Behind the motions, sensations, and thoughts of every animal lies a vast network of cells, the nervous system. The network comprises billions of cells called neurons. A typical neuron is illustrated in Figure 6.47, although neurons with various shapes and sizes make up the total neural system of any animal.

images

Figure 6.47 Sketch of a neuron with inset showing details of a synapse

Neurons specialize in carrying “messages” from one part of the body to another through an electrochemical process that typically causes a voltage spike to travel along the membrane of the neural cell. Messages are received by dendrites, which look like tentacles attached to the cell body. The chemical messages pass through these tentacles into the cell body and then out through one main, long axon. The end of this axon then communicates with dendrites of neurons further down the neural chain, thereby passing messages along from one neuron to the next. Messages between two neurons are usually passed in the form of a chemical flux of so-called neurotransmitters. Excitatory neurotransmitters trigger “go” signals that allow messages to be passed to the next neuron in the communication line. Inhibitory neurotransmitters produce “stop” signals that prevent messages from being forwarded. A single neuron “integrates” incoming signals to determine whether or not to pass the information along to other cells. The activity within a single neuron is typically measured by the rate at which it “fires” voltage spikes.

The simplest model of a population of neurons is the Wilson-Cowan model. It assumes that the entire population of neurons fire at the same rate y (units are number of spikes per millisecond) and are of the same type (i.e., release the same type of neurotransmitters).

Wilson-Cowan Neural Population Model

Let a be the rate at which an external source produces neurotransmitters that stimulate the dendrites of a population of neurons. If a is positive or negative, then the external neurotransmitters are, respectively, excitatory or inhibitory.

Let b be the rate at which each individual neuron releases neurotransmitters when it fires. If b is positive or negative, then the internal neurotransmitters are, respectively, excitatory or inhibitory.

Let c be the rate at which the firing of an active neuron decays exponentially in the absence of external stimulation.

Then the Wilson-Cowan model models the firing rate y (measured in spikes per unit time) of each neuron in the network by the equation

images

Example 5 Modeling memory formation

Consider the Wilson-Cowan model with b = 6 (i.e., the neurons are excitatory) and c = 1 (i.e., in one unit of time the firing rate has dropped by a factor 1/e).

  1. Sketch the bifurcation diagram with respect to parameter a.
  2. Create a plot of y(t) that corresponds to a population of neurons that are initially quiescent—that is, y(0) = 0—and are then subject to an external stimulus that has the following “switching” characteristics: a = −3 for 0 ≤ t < 20 (units of t are milliseconds), a = −1 on 20 ≤ t ≤ 40, and a = −3 on 40 < t ≤ 100.
  3. Discuss the implications of the results.

Solution

  1. Solving for a in terms of y under equilibrium conditions yields:

    images

    Using technology, we obtain the blue curve plotted in Figure 6.48. At the point (a, y) = (−5, 1), we obtain images < 0. Hence, images < 0 in the left region. At the point (a, y) = (0, 0), we obtain images > 0. Hence, images > 0 in the right region. To complete the bifurcation diagram, we draw five phase lines (in red): one at each bifurcation value (i.e., a ≈ −2.5 and a ≈ −3.5) and one to either side of the bifurcation values. Doing so, we obtain the bifurcation diagram illustrated in Figure 6.48.

    images

    Figure 6.48 A bifurcation diagram for the Wilson-Cowan model, where the blue S-shaped curve is the equilibrium solution images = 0 as a function of the parameter a. The red vertical lines are selected phase lines that illustrate how the dynamics depend on a.

  2. We use technology to solve the differential equation (a = −3):

    images

    for 0 ≤ t < 20 with y(0) = 0. This solution is shown below:

    images

    Then, as the domain shifts to 20 ≤ t ≤ 40, we are given that a = −1, so we again use technology to graph a solution of

    images

    Finally, a returns to −3 for the domain 40 < t ≤ 100, and we use from the previous graph an initial value of y(40) = 1, to give the following graph:

    images

    Now we use technology to put these parts together into a single graph, as shown in Figure 6.49.

    images

    Figure 6.49 Graph of how a population of neurons records that it has been subject to a change in background firing rate a on the interval t images [0, 100] (milliseconds)

  3. We see from Figure 6.49 that the population of neurons initially rises from 0 to asymptote at a low firing rate around y = 0.07. In terms of the bifurcation diagram (Figure 6.48), the activity rises from 0 to the equilibrium on the lower arm of the S-shaped equilibrium curve in ya space. When a increases from −3 to −1, the lower arm of equilibria in the bifurcation diagram ceases to exist and the activity rises and approaches an equilibrium close to 1, as seen in Figure 6.49. After “switching back” at t = 40 to the value a = −1, the neural firing rate starts to decline, but now it is only able to drop to the equilibrium on the upper arm of the S-shaped curve in the bifurcation diagram (Figure 6.48). By remaining at the high firing rate, the population of neurons is effectively “remembering” that the background stimulus was in a higher activity state (a = −1) for some period of time before switching back to the lower activity state (a = −3). In this way, the neuron remembers that it was once “switched on.” To clear the memory, the background stimulus a would need to drop below approximately −3.5 (see Figure 6.48).

PROBLEM SET 6.6

Level 1 DRILL PROBLEMS

Draw the phase lines requested in Problems 1 to 6.

images

images

images

Sketch bifurcation diagrams for the equations in Problems 7 to 12.

images

Consider the model

images

of an autocatalytic gene from Example 4. In Problems 13 to 16, two of the parameters are specified. Sketch a bifurcation diagram with respect to the third parameter.

13. k = 1, c = 2 with a as the bifurcation parameter

14. k = 2, c = 1 with a as the bifurcation parameter

15. a = 10, k = 1 with c as the bifurcation parameter

16. a = 10, c = 1 with k2 as the bifurcation parameter

Consider the Wilson-Cowan model for the values of b and c specified in Problems 17 to 22. Sketch the bifurcation diagram with respect to parameter a.

17. b = 5, c = 1

18. b = 4, c = 1

19. b = 8, c = 1

20. b = 4, c = 2

21. b = 8, c = 2

22. b = 12, c = 2

Level 2 APPLIED AND THEORY PROBLEMS

SIS model in Epidemiology

Mathematical epidemiologists often use these symbols: S to denote the number of individuals in a population who are susceptible to a disease, I to denote the number of people infected with the disease, and R to denote the number of individuals who have recovered and are now immune. If no individuals die then the total number of individuals in the population is N = S + I + R. This kind of model is called a SIR model. In the case that all individuals who recover are immediately susceptible, then R = 0 for all time and the model is called an SIS model. Many sexually transmitted infections (STIs)–for example, gonorrhea–do not confer immunity and are best described by SIS models. This is one reason why a single round of antibiotics, even if administered widely on a population basis, will not have a long-term effect in lowering the incidence of STIs.

Let us assume that a susceptible individual encounters and gets infected by infected individuals at a rate proportional to the densities of the product of susceptible and infected individuals in the population. Call this proportionality constant b ≥ 0. The constant b is known as the transmission rate in the epidemiological literature. Also assume that individuals infected with the disease recover from the disease at a constant rate r ≥ 0. Under these assumptions, we obtain this SIS model:

images

Since I + S = N, we know S = NI, so

images

Rearranging terms yields

images

In Problems 23 to 26 sketch a bifurcation diagram with respect to b for r = 1 and with respect to r for b = 1. Discuss under what conditions the disease persists in a population confined to living in a group of indicated size.

23. N = 1,000 (boarding school)

24. N = 10,000 (army camp)

25. N = 100,000 (isolated town)

26. N = 1,000,000 (isolated city)

Habitat destruction

Consider a population living in a patchy environment. Let y be the fraction of patches occupied by the species of interest. Let c ≥ 0 denote the colonization rate (i.e., the rate at which individuals from one patch colonize an empty patch), d ≥ 0 the rate at which individuals clear out of a patch, and 0 ≤ D ≤ 1 the fraction of patches destroyed by humans. Then we get the following model (developed by Harvard biologists, Richard Levins and David Culver*):

images

Sketch the bifurcation diagram for this differential equation for the information given in Problems 27 to 32.

27. Assume that D = 0. Sketch bifurcation diagrams for d when c = 1 and for c when d = 0. Under what conditions does the population persist?

28. Assume that D = 0. Sketch bifurcation diagrams for d when c = 2 and for c when d = 1. Under what conditions does the population persist?

29. Assume that D = 0.5. Sketch bifurcation diagrams for d when c = 1 and for c when d = 0.5. Under what conditions does the population persist?

30. Assume that D = 0.5. Sketch bifurcation diagrams for d when c = 2 and for c when d = 2. Under what conditions does the population persist?

31. Assume that c = 3/2. Sketch bifurcation diagrams for D when d = 1/2 and for d when D = 0. Under what conditions does the population persist?

32. Assume that d = 2. Sketch bifurcation diagrams for D when c = 4 and for c when D = 1/2. Under what conditions does the population persist?

Lotka-Volterra predation

In the 1920s two mathematicians, Vito Volterra (1860–1940) and Alfred Lotka (1880–1949) considered models of the density of a prey species, denoted here by the variable x, predated by a species at a density denoted here by the variable y. They wrote two differential equations, one for the prey and one for the predator, in which the prey equation included the predator density and the predator equation included the prey density. In Chapter 8 we will explore how to analyze a system of two interdependent differential equations. Here, however, with the methods covered in this chapter, we can analyze the behavior of the prey equation or the predator equation where the density of the other species appears as a parameter. The general form of the prey equation is

images

where g(y) is a per capita growth rate of the prey species and h(x) is the rate at which each unit of predator is able to extract prey. Note it is assumed that both g(0) = 0 and h(0) = 0. The general form of the predator equation is

images

where h(x) is the prey extraction rate per predator appearing in the prey equation, 0 < b < 1 is the efficiency with which predators can convert a unit of consumed prey into their own biomass (ingestion, digestion, metabolism, etc.), and f(y) is the rate at which predators die when they have no prey species to feed upon.

In Problems 33 to 35 sketch the bifurcation diagram for the specified growth and extraction functions in the prey equation in which the density y of the predators is regarded as a parameter in the prey equation, and in Problems 36 to 37 sketch the bifurcation diagram for the specified extraction and mortality functions in the predator equation in which the density x of the prey species is regarded as a parameter.

33. In one form of the Lotka-Volterra model, g(x) is a decreasing linear function and h(x) is proportional to x. Consistent with these assumptions, let g(x) = 0.5 images and h(x) = x. Determine under what conditions the population persists.

34. In another form of the Lotka-Volterra model, g(x) is constant and h(x) is an increasing and saturating function of prey density x. Consistent with these assumptions, let g(x) = 0.5 and h(x) = images. Under what conditions is the prey population extinction bound?

35. Assume g(x) = 0.5 images and h(x) = images. Under what conditions is the prey population driven to extinction by predation?

36. Assume b = 0.2 and f(y) = 1. Under what conditions does the predator population persist when h(x) = x?

37. Assume b = 0.2 and f(y) = y. Under what conditions does the predator population persist when h(x) = x?

38. A self-regulatory genetic network. Smolen and colleagues* investigated a model of a single transcription factor, TF-A, that activates its own transcription TF-A by first forming a homodimer, which then activates transcription by binding to enhancers (TF-REs). A rapid equilibrium is assumed between monomeric and dimeric TF-A. The transcription rate saturates with TF-A dimer concentration to a maximal rate a, which is proportional to TF-A phosphorylation. Responses to stimuli are modeled by varying the degree of TF-A phosphorylation. A basal synthesis rate d is present, as well as a first-order process for degradation, −cy. If y denotes the concentration of TF-A then the model is given by

images

Assume that b = 1, c = 1, and d = 0.1. Sketch a bifurcation diagram over the region 1 ≤ a ≤ 3 and 0 ≤ y ≤ 3. Discuss when you expect to see two stable equilibria.

39. Evolution of cooperation, part 3. Problem 31 in Section 6.5 investigated how individuals who interact frequently and respond to the strategy of their opponents can promote the evolution of cooperation. If opponents interact on average n times and cooperation gives a benefit B to the opponent and a cost C to the cooperator, then the payoff matrix for the tit-for-tat and defect strategies are as shown here:

images

  1. Write a replicator equation for this game.
  2. Assume B = 4 and C = 3 and sketch a bifurcation diagram with respect to the parameter n.
  3. Discuss the implications for the evolution of cooperation.

40. Suppose the growth rate of a whale population at density N (individuals per million square kilometers of ocean), harvested at a rate h, is given by

images

where the units of t are years.

  1. Sketch a bifurcation diagram with respect to the parameter h as it varies over the interval [0, 8].
  2. If h = v N, then sketch a bifurcation diagram with respect to the parameter v as it varies over the interval [0, 0.12].

CHAPTER 6 REVIEW QUESTIONS

  1. Determine for what values of a, b, and c, y(t) = et + at2 + bt + c is a solution to t2 + images = y.
  2. Find the general solution to the differential equation images = y tan t.
  3. In Ludwig von Bertalanffy's classic paper on modeling the growth of individuals, he modeled the growth of a species of guppy (Lebistes reticulatus) using the differential equation

    images

    where L(t) is the length of the guppy in millimeters in week t. The data and the best-fitting version of the model are shown in Figure 6.50. Given that the length at one week is 12.5 mm and at two weeks is 15.6 mm, and that the limiting length is 26.1, find L(t).

    images

    Figure 6.50 Growth of the guppy species Lebistes reticulatus with the best-fitting von Bertalanffy growth curve.

  4. Let images = f(y) where the graph of f(y) is as shown here:

    images

    Sketch the phase line for this differential equation and sketch solutions of this differential equation with initial conditions y(0) = −0.1 and y = 0.1.

  5. Find the implicit solution to the initial value problem lem yimages = et+2y sin t where y(0) = 0.
  6. Cooperation can be observed in natural populations. To better understand under what conditions it may evolve, imagine that each time two opponents meet they interact two times. During these interactions, individuals play one of two strategies: defect always or tit-for-tat, in which case an individual initially cooperates but switches to defecting if the opponent defected. If the benefit of interacting with a cooperating individual is B and the cost of cooperating is seven, then the payoff matrix is given by

    images

    1. Write a replicator equation for this game where y is the frequency of individuals playing the tit-for-tat strategy.
    2. Linearize the equation at the y = 1 equilibrium and determine for what values of B this equilibrium is stable.
    3. Discuss the implications for the evolution of cooperation.
  7. Sketch the bifurcation diagram for the differential equation images = ry+y3 with respect to the parameter r.
  8. Estimate a solution to the differential equation images = 2t(t2y) with y(0) = 3 over the interval 0 ≤ t ≤ 2 using Euler's method with step size h = 0.4.
  9. The radioactive substance gallium-67 (symbol67Ga) used in the diagnosis of malignant tumors has a half-life of 46.6 hours. If we start with 100 mg of67 Ga, what percentage is lost between the 30th and 35th hours? Is this the same as the percentage lost over any other five-hour period?
  10. Sketch the solution to the initial value problem images = f(t, y) with y(0) = 0.25 for the slope field shown here:

    images

  11. A population of animals on Catalina Island is limited by the amount of food available. Suppose it is shown that there were 1,800 individuals present in 1980 and 2,000 in 1986, and studies also suggest that 5,000 individuals can be supported by the conditions present on the island. Use the logistic model images = rN(1 − N/K) to predict the size of the population in the year 2000.
  12. In the 1960s, research scientist Anna Laird for the first time successfully used the Gompertz model to fit data of growth of tumors. Recall the Gompertz model is given by

    images

    where y is the size of the tumor, t is measured in hours, K is the theoretical limiting size of the tumor (never achieved), and α is a positive constant. For one of the rat data sets used by Laird, the growth of the rat's tumor was modeled using the parameter estimates α = 0.02, y(0) = 0.4 grams, and K = 9,600 grams. Find the size of the tumor after t hours.

  13. A lake has a volume of 6 billion ft3, and its initial pollutant content is 0.22%. A river whose waters contain only 0.06% pollutants flows into the lake at the rate of 350 million ft3/day, and another river flows out of the lake also carrying 350 million ft3/day. Assume that the water in the two rivers and the lake is always well mixed. How long does it take for the pollutant content to be reduced to 0.15%?
  14. The velocity v of a skydiver falling to the ground is governed by the equation images = mgkv2, where m is the mass of the skydiver, g is acceleration due to gravity, and k > 0 is a constant proportional to air resistance. Plot the phase line for this equation and find the terminal velocity images v(t).
  15. Consider the differential equation images = ayy2. Verify that y = 0 is an equilibrium and use linearization to determine for what a values 0 is stable or unstable equilibrium.
  16. A refined model of harvesting a logistically growing population is given by

    images

    where N is the population abundance in thousands of individuals, r is the population's intrinsic rate of growth in the absence of harvesting, K is the carrying capacity of the population in the absence of harvesting, H is the harvesting effort, and a > 0 is the half saturation constant for harvesting. Assume r = 0.05, K = 100, and a = 1.

    1. Sketch a bifurcation diagram for N ≥ 0 and H ≥ 0.
    2. What is the maximal harvesting effort H that is sustainable?
  17. Spruce budworm is a serious insect pest in eastern Canada. In years with large outbreaks, these insects can defoliate and kill a major portion of balsam firs in a forest within four years. Mathematical biologist Donald Ludwig and colleagues developed an elegant model of spruce budworm and forest dynamics. For the budworm dynamics, they assumed that the population exhibited logistic growth and experienced bird predation. Bird predation was modeled using a type III functional response. Hence, the dynamics are given by

    images

    Assume K = 15, b = a = 1.

    1. Sketch a bifurcation diagram with respect to r. Hint: Solve for r in terms of N and use technology to graph the resulting curve.
    2. Experimental observation suggests that for a young forest r < 0.2. As the forest matures, r slowly increases to 1. Discuss what increasing r slowly implies about the abundance of spruce budworms.
  18. A patient requires a concentration of 2 mg per liter of a drug in his bloodstream. The clearance rate of the drug is 0.2 per hour (i.e., a half-life of ≈ 3.5 hours). The patient has five liters of blood and initially has no drug in his bloodstream.
    1. Find the infusion rate required to get a concentration of 2 mg per liter in two hours.
    2. Find the infusion rate required to maintain a concentration of 2 mg per liter in the long term.
  19. Consider

    images

    1. Sketch the slope field for this differential equation.
    2. Use Euler's method with Δt = 0.5 to approximate the solution satisfying y(0) = 0 over the interval 0 ≤ t ≤ 1.5.
    3. Sketch the numerical solution over your slope field and briefly discuss whether the approximation is reasonable.
  20. Consider the initial value problem

    images

    1. Find the solution to the initial value problem.
    2. Find images y(t) for the answer you found in part b.

GROUP PROJECTS

Seeing a project through on your own, or working in a small group to complete a project, teaches important skills. The following projects provide opportunities to develop such skills.

Project 6A Modeling Diseases

In Problem Set 6.6 we explored the behavior of epidemics in a population of size N individuals. We separated these individuals, into susceptible persons, with numbers denoted by S, and infected persons, with numbers denoted by I. In our differential equation model for epidemics, infected individuals once recovered were immediately susceptible, and the relationship N = S + I held throughout; that is, no new individuals entered the population (births or immigration) and no individuals left (deaths or emigration). We also raised but did not explore the possibility that individuals can be removed from the epidemic process either by recovering and becoming immune or by dying. Let the number of individuals in this removed class be represented by R.

The goal of this project is to address epidemiological questions such as these: If an infected individual is introduced into a population of susceptible individuals will an epidemic occur? If it does occur, how many people will ultimately catch the disease?

Begin with the assumptions that the disease under consideration confers permanent immunity on any individual who has completely recovered from it, and that the disease has a negligibly short incubation period. This latter assumption implies that an individual who contracts the disease becomes infected immediately afterward.

To complete this project, perform the following tasks:

  • Write a system of three first-order differential equations based on the following additional assumptions:

    Assumption 1: The total population remains fixed at a level N in the time interval of consideration.

    Assumption 2: The rate of change of the susceptible population is proportional to the product of the number of susceptible and the number of infected.

    Assumption 3: Individuals from the infected class are removed and enter the removed class at a rate proportional to the number of infected.

  • Assume that R(0) = 0. Use the fact that S(t) + I(t) + R(t) = N, and special features of the images and images equations, to show that there exists a function F(R) such that images = F(R)—that is, the system reduces to one first-order equation. Hint: Use chain rule to write dS/dR and solve the resulting differential equation to express S as a function of R.
  • Show that images = F(R) can be rewritten as

    images

    with a ≥ 1 and b > 0 by appropriately rescaling R and t to x and τ.

  • Determine the number of fixed points of equation 6.1 and classify their stability.
  • Show that if b < 1 and x(0) = 0, then x′(τ) is increasing from τ = 0 until it reaches a maximum at some time τmax > 0. Show that if b > 1 and x(0) = 0, then x′(τ) is decreasing. What do these facts imply about I(t)? Discuss the biological implications.

Project 6B Save the Perch Project*

Happy Valley Pond is currently populated by yellow perch. A map of the pond is shown in Figure 6.51. This map shows at each grid point the depth of the pond in feet when the dam is at spillover level. Assume that each grid cell is 5 × 5 ft2. Use this map to estimate the number of gallons of water in the pond when the water level is exactly even with the top of the spillover dam. This information will be needed to construct your model to account for the following additional facts.

images

Figure 6.51 Happy Valley Pond is fed by two springs

Water flows into the pond from two springs and evaporates from the pond at rates given in the following table.

images

At all times the pond is well mixed by the inflows, outflows and wind. Recently, spring B became contaminated by an underground salt deposit so that its water is a 10% salt solution, which means that 10% of a gallon of water from spring B is salt. Assume that the salt does not evaporate but is instead well mixed with the water in the pond so that the rate of salt lost is determined by the outflow rate of water and the well-mixed concentration of salt in the pond at the time of outflow.

The yellow perch in the pond are salt intolerant and start to die when the concentration of salt exceeds 1%. There was no salt in the pond before the contamination of spring B. You and the members of your group (if you have one) have been called upon by the Happy Valley Bureau of Fisheries to try to save the perch. Unfortunately, spring B is underground and cannot be capped off, but fresh water can be piped from other sources to help dilute the salt concentration in the pond.

Assume the salt contamination in spring B started at the beginning of the dry season in 2004 (t = 0), when the pond was exactly even with the top of the spillover dam. Selecting the units of t to be hours, formulate a differential equation for the amount of salt in the pond at any time t after the start of the dry season in 2004. Remembering to take into account the seasonal nature of the flows. Using differential equation solving technology, draw a graph of the amount of salt in the pond over the dry season in 2004 and over the following wet season.

Now use the model to address the following questions (assume no management interventions unless specifically asked to consider them):

  1. What is the equilibrium solution under persistent dry season conditions?
  2. What is the equilibrium solution under persistent wet season conditions?
  3. What will the salt profile look like in the long run over a combined dry and wet season? (Use your model to produce the profile for the first dry-wet season, the second dry-wet season, and so on until all consecutive profiles are identical to the desired number of decimal places.)
  4. From this long-term profile, identify the periods when the perch are threatened (i.e., salt concentrations exceed 1%) and calculate how much water will need to be piped in to ensure that the perch remain safe. Whenever fresh water is piped in, it is always at the rate of 100 gallons of pure water per hour (all the Happy Valley Bureau of Fisheries Management Council can decide is when to switch the spigot on and off).

In your report include the design and analysis of a plan that can be used to ensure that the water in the pond never gets too salty for the perch. Can you come up with any interesting innovations that might help manage the salinity of the pond?

*Colin W. Clark, Mathematical Bioeconomics: The Optimal Management of Renewable Resources, (New York: John Wiley & Sons, 1976).

*From D. N. Burghes, I. Huntley, and J. Mc-Donald, Applying Mathematics: A Course in Mathematical Modeling (Halsted Press, 1982).

*Data source: M. D. Sabbath, W. C. Boughton, and S. Easteal, “Expansion of the Range of the Introduced Toad Bufo marinus in Australia from 1935 to 1974, Copeia 3 (1981): 676–680.

*H. von Foerster, P. M. Mora, L. W. Amiot, 1960. “Doomsday: Friday, 13 November, A.D. 2026,” Science, Vol. 132 no. 3436 pp. 1291–1295 DOI:10.1126/science.132.3436.1291.

*A. S. Perelson, A. U. Neumann, M. Markowitz, J. M. Leonard, and D. D. Ho, “HIV-1 Dynamics In Vivo: Virion Clearance Rate, Infected Cell Lifespan, and Viral Generation Time,” Science 271 (1996): 1582–1586, and A. S. Perelson and P. W. Nelson, “Mathematical Analysis of HIV-1 Dynamics In Vivo,” SIAM Review 41 (1999): 3–44.

*R. Levins and D. Culver, Regional coexistence of species and competition between rare species Proceedings of the National Academy of Sciences 68 (1971), pp. 1246–1248.

*P. Smolen, D. A. Baxter, and J. H. Byrne. Frequency, selectivity, multistability, and oscillations emerge from models of gene networks, Am. J. Physiol., 274 (1998), pp. C531–C542.

*Reprinted with permission from Stephen Hilbert, Diane D. Schwartz, Stan Seltzer, John Maceli and Eric Robinson, Calculus: An Active Approach with Projects (MAA 2010). Copyright the Mathematical Association of America 2013. All rights reserved.

..................Content has been hidden....................

You can't read the all page of ebook, please click here login for view all page.
Reset